129
SECTION II: GRAMMAR AND VOCABULARY (7 points) PART 1: Choose the option (A,B,C or D) that best completes each of the following sentences. Write your answers in the numbered table below. 1. Our country is rich ________ natural resources. A. of B. with C. about D. in 2. The idea to ________ a visit to the local council residence was welcomed by all the visitors. A. pay B. do C. go D. walk 3. In his anxiety to make himself________, he spoke too loudly and too slowly. A. understand B. understanding C. understood D. to understand 4. ________ for the fact that he was working abroad, he would willingly have helped with the project. A. If it hadn’t been B. If it had been C. Had it been D. Hadn’t it been 5. Some animals are on the ________ of becoming extinct. A. edge B. verge C. side D. tip 6. The play is very long but there are three ________ A. intervals B. breaks C. rests D. naps 7. The last lecture ________ completely over my head. A. got B. went C. was D. left 8. Could I pick your ________ on the subject before the meeting? A. brains B. head C. intellect D. mind 9. I was prepared to lend my brother some money but he turned ________ my offer. A. back B. up C. out D. down 10. I ________ with the performances but I got flu the day before. A. was to have helped B. helped C. was to help D. had helped 11. The dying man’s speech was so________ that no one was able to interpret his last request. 1

1. on hsg lop 12

Embed Size (px)

DESCRIPTION

 

Citation preview

SECTION II: GRAMMAR AND VOCABULARY (7 points)

PART 1: Choose the option (A,B,C or D) that best completes each of the following sentences. Write your answers

in the numbered table below.

1. Our country is rich ________ natural resources.

A. of B. with C. about D. in

2. The idea to ________ a visit to the local council residence was welcomed by all the visitors.

A. pay B. do C. go D. walk

3. In his anxiety to make himself________, he spoke too loudly and too slowly.

A. understand B. understanding C. understood D. to understand

4. ________ for the fact that he was working abroad, he would willingly have helped with the

project.

A. If it hadn’t been B. If it had been C. Had it been D. Hadn’t it been

5. Some animals are on the ________ of becoming extinct.

A. edge B. verge C. side D. tip

6. The play is very long but there are three ________

A. intervals B. breaks C. rests D. naps

7. The last lecture ________ completely over my head.

A. got B. went C. was D. left

8. Could I pick your ________ on the subject before the meeting?

A. brains B. head C. intellect D. mind

9. I was prepared to lend my brother some money but he turned ________ my offer.

A. back B. up C. out D. down

10. I ________ with the performances but I got flu the day before.

A. was to have helped B. helped C. was to help D. had helped

11. The dying man’s speech was so________ that no one was able to interpret his last request.

A. incoherent B. indiscreet C. nonchalant D. impotent

12. Very soon I found some other people to ________ and we began to write songs.

A. keep up with B. team up with C. talk through with D. get along with

13. ________ chair the meeting.

A. John was decided to B. It was decided that John should

C. There was decided that John should D. John had been decided to

14. I thought about the problem but I couldn’t ________ a solution.

A. come in for B. come across C. come up with D. come out

15. ________, they slept soundly.

A. Hot though was the night air B. Hot though the night air was

C. Hot as was the night air D. Hot although the night air was

1

PART 2: Read the text below and think of the word that best fits each gap. Use only ONE word in each gap. There

is an example at the beginning (0). Write your answers in the numbered table below.

Is your school just as you wanted (0)_______ to be? Or are there things you and your classmates (1) ________

change, given the opportunity? This is your chance to express your ideas about (2) ________ the ideal school is like.

Our competition is open to (3) ________ student between the ages of twelve and eighteen. You can enter (4)

________ an individual or your whole class can work together on a team entry. Your entry can take any form – a

piece of writing, a picture, or even architectural plans. It is

completely (5) ________ to you. What we are looking for is evidence (6) ________ originality, imagination and,

above (7) ________, the genuine views of young people.

By (8) ________ part in this, you will help in a study being carried out at a leading university. All work entered (9)

________ the competition will be kept at the university and used in research. Entries cannot be returned (10)

________ of this. But it also means that, even (11) ________ you do not win, your views will still be heard and will

remain for future educationalists to study.

Entries must reach us no (12) ________ than Friday 30 April. Winners will receive valuable prizes of computer

equipment and software for their schools.

PART 3: Read the text below and look carefully at each line. Some of the lines are correct and some have a

word which should not be there. If a line is correct, put a tick (√). If the line has a word that should not be

there, write the word at the end of the line. There are two examples at the beginning.

0 Congratulations on winning of the tennis championship! You must be of

00 very pleased, especially since the prize is quite a lot of money. √

1 What are you going to spend it on? You could even buy a new car

2 with all that money! You should have be in great shape after all the

3 training you have been doing. It must be so very hard work, practising

4 all those hours for every day but it is worth it in the end, isn’t it?

5 Perhaps you are thinking of going on holiday so that you can have

6 a break from tennis and relax. Can you tell me exactly what is kind of

7 tennis racket you chose for the competition? If I would get the same,

8 it might help me to improve my game. Anyway, congratulations on

9 your great victory! I’m still studying English every single day and the

10 course has three months to go. I have moved house, as if you can see

11 from my new address. Make sure you reply back to the right address!

12 Your last letter went to my old address, but it wasn’t by your fault

13 because I hadn’t told anyone who I had moved then. Did you know

14 that I have had a job for the last three weeks? I work in a restaurant

15 four evenings a week. I like it, but I don’t arrive at home until one

o’clock in the morning, which is a bit inconvenient.

2

PART 4: Supply the correct tense or form of the verbs in brackets to complete the passage. Write your answers in

the numbered table below. (0) has been done as an example.

18,000 years ago, much of Europe (0) ____lay____(lie) (1)________(bury) beneath vast sheets of ice, hundreds of

metres thick. Ever since this astonishing fact (2)_______(discover) in the last century, scientists (3)

________(speculate) on the nature of the Ice Age climate, and the circumstances that brought it to an end.

More recently, people (4) ________(wonder) if climatic changes could (5)________(take) place in our own time.

During the early 1970s there (6)_______(be) disastrous droughts in Africa, and frequent failures of Indian monsoon.

In 1976, Europe sweltered in the hottest summer for over a century, and (7)_______(experience) one of the worst

droughts since records began. Could such events as these be symptoms of a worldwide climatic shift?

Even small changes in climate that (8)________(occur) from time to time can have a highly damaging effect on

agriculture. With food reserves now (9)________ (stand) at only a few per cent of annual production, the world is

extremely vulnerable (10) ________(adverse) shifts in climate. It is therefore vitally important for us to understand

how climate changes take place.

PART 5: Think of ONE WORD only which can be used appropriately in all three sentences. Write your answers in

the numbered table below. (0) has been done as an example.

0. She screamed for _______ and luckily someone heard her.

Thanks for all your _______ through such a difficult time.

Practical ________ is offered through our accommodation service to new students.

1. I get off at the next _______.

The referee was forced to _______ the game because of heavy snow.

Work has temporarily come to a _______ while the funding in reviewed.

2. Keep your _______ on your work.

Do you have anyone in _______ for this job?

Don’t _______ her. She didn’t mean what she said.

3. There is a little gift _______ around the corner.

I do a weekly _______ at the supermarket.

He didn’t expect his own mother to _______ him to the police.

4. Everyone wished her the best of _______ at university and hoped she would enjoyed it.

Jenny won the competition at her first attempt – perhaps it was beginner’s _______!

There is no such thing as _______, we are capable of creating our own good fortune.

5. Politicians can abuse their position of _______ .

The _______ supply to our house was cut because of roadwork.

I’m afraid I do not have the _______ to authorize this change.

6. Few people could have predicted the huge impact of information _______ .

No matter how advanced _______ becomes, machine will never be able to think like humans.

It’s a waste of time for humans to do tasks that modern _______ can do.

7. She was struck by the sudden _______ that he might already have left.

3

He dived in after her without a second _______.

It was once _______ that the sun traveled around the earth.

8. Governments should give as much foreign _______ as possible to poorer countries.

In certain circumstances, emergency _______ in the form of money should be sent immediately.

The most successful long-term _______ programmes encourage self-help.

Write your answers here:

0. help 1. 2.

3. 4. 5.

6. 7. 8.

PART 6: For questions from 1-10, read the text below. Use the word given in capitals at the end of some of the

lines to form a word that fits in the gap in the same line. Write your answers in the numbered table below. There is

an example at the beginning (0).

Before going to an interview, it is (0)_________ to go through a mock interview.

This will give you the opportunity to try out your technique and answers live. It is

also a chance to receive feedback that is (1)__________ in guiding you towards

improving your interview style and general (2) _________ Just one mock interview

will result in a (3)_________ improvement in your interview skill. Why? For the

same reason that a (4) _________ doesn’t exist while it is still on paper or floating

in your head. It only exists when you give it (5)_________ The first time you give it

in front of an audience, it will come out nothing like the one you prepared.

It is the same with being interviewed. It is not enough to look at a question and say,

‘Yeah, I know the answer to that one.’ You need to practise your answer live; this is

not the time to talk to yourself in front of a mirror. Seek out a (6) _________ and

have the session videotaped. Then you will have two opinions – the interview’s and

your own. You will find you get a completely different (7)________ when listening

to yourself than when you are watching yourself saying something. Just as your

voice always sounds different on tape, so do your (8) _______. You will be glad the

image is captured on tape and not in a potential employer’s mind. For maximum

effect, you should (9)________ your answers and go through a second mock

interview. This should help with any (10)________ and give you more confidence

for the real interview.

ADVISE

BENEFIT

PRESENT

NOTICE

SPEAK

ORAL

PROFESSION

IMPRESS

RESPOND

VISIT

EASE

SECTION III: READING (4 points)

PART 1: Read the article below and decide which answer (A,B,C or D) best fits each gap. Write your answers in

the numbered table below. There is an example at the beginning (0).

4

Students and jobseekers keen to get onto the course or into the workplace of their (0)________ hope that voluntary

work will help them (1)________ from the crowd. This chance to (2)________ experience – personally and

professionally – is (3)________ on the wish-list of young people.

A survey carried out last year revealed that young and old (4)________ said volunteering had improved their lives,

particularly those (5)________in conservation or heritage work.

Businesses recognise its importance and get to (6)________their profile in the community, while staff get a break

from their daily routine to develop ‘soft skill’, (7)________initiative and decision-making. One volunteering

organisation is (8)________ another survey to find out if volunteering does make a difference in the workplace, or if it

is something businesses do simply to improve their (9)________ .

Not (10)________are business-sponsored placements becoming more common, the government is also investing

money and aiming to (11) ________volunteers. The push is clearly on to make volunteering as attractive as possible

to everyone.

And the more people who participate, the more the act fulfils its (12) ________of making the world a better place.

0. A. alternative B. choice C. option D. election

1. A. stand out B. lift out C. pick out D. point out

2. A. win B. achieve C. collect D. gain

3. A. extreme B. high C. sharp D. strong

4. A. similar B. the same C. alike D. too

5. A. committed B. associate C. connected D. involved

6. A. raise B. increase C. arouse D. motivate

7. A. such B. such as C. such like D. such and such

8. A. governing B. guiding C. conducting D. directing

9. A. representation B. look C. image D. figure

10. A. only B. just C. merely D. simply

11. A. claim B. recruit C. bring D. enter

12. A. aim B. direction C. mark D. design

PART 2: Look at the sentences below about a hotel. Read the text to decide if each sentence is correct or incorrect.

If it is correct, write C. If it is incorrect, write I.

Write your answers in the numbered table below.

1. During the 1980s, few tourists used to go to the Arctic in summer.

2. People came in large numbers to Jukkasjärvi to see the Arctic Hall.

3. The artist encouraged people to sleep in the Arctic Hall.

4. Each winter, guests come and sleep in the hotel before it is finished.

5. Progress when building the hotel is influenced by the weather.

6. The temperature inside the hotel changes according to the temperature outside.

7. Some clothes are provided by the hotel.

8. Guests should buy boots which fit as tightly as possible.

9. Items ordered through the ICEHOTEL shop will be delivered to your home.

5

10. It is possible to take a train from the airport to the IceHotel.

THE ICEHOTEL

For many years the Arctic was a popular destination in the summer season to see the land of the midnight sun but in

winter the few inhabitants had the snow and ice to themselves. By the end of the 1980s it was decided that the dark

and cold winter should be seen as an advantage. In the winter of 1990 the French artist Jannot Derit was invited to

have the opening of an exhibition in a specially built igloo (a building made of snow) in the little town of Jukkasjärvi

on the frozen Torne River. The building, named Arctic Hall, attracted many interested visitors to the area. One night a

group of foreign guests decided it would be a good idea to sleep in the Arctic Hall. The following morning the brave

group were very pleased with their experience and the idea of an ice hotel was born. Today it is world famous.

As soon as winter begins, a team of snow builders, architects and artists from all over the world come to Jukkasjärvi

and they make the hotel for that year. As one part is completed, it opens to visitors and overnight guests, while the

other parts are still being built. The first part is completed in December and each week after that a new part opens,

until January 7th when the hotel is completed. As the ICEHOTEL is built under the open sky, using the natural

materials of the winter season, the finishing date depends on nature and therefore there are sometimes changes to the

plan. In the spring, as the weather gets warmer, the hotel melts.

Inside the hotel, the temperature is never colder than –5 °C to –8 °C, however cold it may be outside. Winter outer

clothes such as warm overalls, hats and gloves are included in the cost of guests’ stay at the hotel. In addition to this,

it is a good idea for guests to bring sweaters and a scarf as well as plenty of woolen socks and to choose footwear that

is larger than normal to allow space for thick socks.

If you are planning to come to the hotel, you can buy warm sweaters, woolen socks and much more on the ICEHOTEL

website. You can order these and the equipment you will need at the same time as you book your visit. The items will be

delivered to your room when you check in.

The hotel is in the village of Jukkasjärvi, 200 km above the Arctic Circle but only 15 km from Kiruna airport and 17

km from Kiruna train station. Transport by bus can be arranged from the airport or train station to the IceHotel.

PART 3: Read the following passage and do the tasks below. Write your answers in the numbered table.

The Atmosphere of Venus

Venus, also called the Morning Star and Evening Star, is the second-closest planet to the sun and the brightest

object in the night sky. The planet orbits the sun every two hundred and twenty four Earth-days and is sometimes

referred to as Earth’s sister planet because the two share both a similar size and bulk. What is not similar, however, is

Venus’s atmosphere in comparison to Earth’s atmosphere.

The atmosphere on Venus is much heavier and has a higher density than that of Earth. Venus’s atmosphere

also expands significantly higher than Earth’s atmosphere although a thick cloud cover makes the surface of Venus

nearly impossible to see unless observed through radar mapping.

While the pressure and temperature of Venus’s upper atmosphere are comparable to those of Earth, the heat

and pressure of the lower atmosphere are not unlike a furnace. Venus’s atmosphere is very thick due to a composition

consisting mainly of carbon dioxide, and a small amount of nitrogen. If man could survive the extreme heat of

6

Venus’s surface (400 degrees Celsius), then he would have to contend with a surface pressure that is more than

90 times that of Earth. Venus’s extremely high temperature is thanks to the greenhouse effect caused by such a large

amount of carbon dioxide. The greenhouse effect is a process by which the sun’s infrared radiation is more readily

absorbed by the atmosphere. Just like in a real greenhouse used to grow plants years round, the proliferation of carbon

dioxide traps radiation and warms Venus’s atmosphere. Due to this phenomenon, Venus boasts a higher atmospheric

temperature than Mercury, even though Venus is twice the distance from the sun.

However, scientists postulate that Venus’s atmosphere was not always so hot. [A] Studies show that large

bodies of water were once on Venus’s surface but that eventually evaporation of all the water caused the runaway

greenhouse effect which regulates the planet today. [B] Thus Venus has become a critical study for today’s scientists,

as human being are only beginning to struggle with the early stages of the greenhouse effect. [C] Our problems do not

stem from evaporated water supplies but from a propagation of carbon dioxide and other greenhouse gases due to

industrial and automobile emissions. [D]

Another interesting characteristic to note regarding Venus’s atmosphere is that its daytime temperatures and

nighttime temperatures are not that far removed from each other. This is due to the thermal inertia, the ability of a

substance to store heat despite changing temperatures and the transfer of heat by Venus’s strong winds. Although

winds on the surface of Venus move slowly in comparison with Earth’s winds, Venus’s air is so dense that a slow-

moving there can move large obstructions and even skip stones along the planet’s surface.

In 1966, humankind made its first attempt at sending a recording instrument into Venus’s atmosphere. The

Venera 3 probe did collide with Venus surface; however, the abrupt impact caused its communication system to fail,

and it was unable to send and feedback. In 1967, Venera 4 successfully enter Venus’s atmosphere and was able to

take many readings, one of which recorded that Venus’s atmosphere was between ninety and ninety-five percent

carbon dioxide. Subsequent Venera probes were sent into Venus’s atmosphere, but most of them succumbed to the

crushing air pressure.

Questions 1-7: Choose the answer (A, B, C or D) which you think fits best according to the passage.

1. According to paragraph 1, Venus is named the Morning Star and Evening Star because

A. it is very bright

B. it is close to the sun

C. it can be seen from evening till morning

D. it is used to find the direction by sailors

2. The word that in paragraph 2 refers to

A. size B. bulk C. atmosphere D. density

3. Which of the following best expresses the essential information in the bold sentence in paragraph 3? Incorrect

answer choices change the meaning in important ways or leave out essential information.

A. Earth experiences greater surface pressure than Venus.

B. If a man could survive its surface pressure.

C. The surface pressure and heat of Venus are much greater than those on Earth.

D. Venus’s surface temperature and pressure make it uninhabitable by humans.

7

4. According to paragraph 3, the greenhouse effect on Venus is owed to

A. the small amounts of nitrogen

B. the rapid increasing amounts of carbon dioxide

C. growing plants

D. the high atmospheric temperatures

5. In paragraph 4, the author of the passage implies that Earth

A. might suffer the same greenhouse effect as Venus

B. once had an atmosphere similar to Venus’s

C. has bodies of water similar to those on Venus today

D. is experiencing a reduction of carbon dioxide emissions

6. Look at the four blanks […] in paragraph 4 that indicate where the following sentence could be added to the

passage.

Although the causes are different, the ramifications are the same.

Where would the sentence best fit?

7. The word propagation in paragraph 4 is closest in meaning to

A. generation B. elimination C. evaporation D. desecration

Question 8: Complete the brief summary of the passage by selecting the THREE answer choices that express

important ideas in the passage. The introductory sentence for the summary is provided bellowed.

Scientists look at Venus to predict Earth’s future.

●…………………………………………………

●…………………………………………………

●…………………………………………………

Answer Choices

A. Venus once had large bodies of water that elaborated and cause a rapid increase in carbon dioxide.

B. Earth’s wind has a greater velocity than Venus’s because the air movement on Venus is denser and can even

more large obstructions.

C. Spaceships landing on Venus, though often crushed by Venus’s atmosphere, have revealed much about its

carbon dioxide filled atmosphere.

D. If man could survive the hot temperature of Venus, then he would have to contend with the great surface

pressure.

E. The first space probe of Venus was made in 1966.

F. Scientists are concerned that conditions on Earth that propagate significant quantities of carbon dioxide will

produce a greenhouse effects similar to Venus’s.

Write your answers here:

1. 2. 3. 4.

5. 6. 7. 8.

8

PART 4: You are going to read a magazine article about learning how to fly a plane. Eight paragraphs have been

removed from the article. Choose from paragraphs A-I the one that fits each gap (1-8). There is one extra

paragraph that you do not need to use. Write your answers in the numbered table below.

Learning to Fly

I had been testing cars and motorcycles for over twenty years. I couldn’t take any more. It wasn’t terribly exciting

and, in any case, new cars were beginning to look identical and drive similarly. What I needed was a new challenge.

1

Unfortunately, I wore glasses. The Royal Air Force wouldn’t consider anyone for pilot training unless they had

perfect eyesight. Halfway through an aptitude test, they realised that my eyes were far from perfect. I didn’t stand a

chance.

2

It was an obvious choice. It’s just twenty minutes’ drive from my home. It’s very quiet, too, so the £90 per hour for

the training is spent flying in the air, not waiting on the ground for other planes to take off.

3

It took me a whole year to get my private pilot’s license. It started well, with my first solo flight coming after just

seven hours. Then came all the studying, the exams, the hard work. I never thought I’d get to the end of it.

4

Then came last winter and the end of the course was in sight. For weeks, the weather was so terrible that for most of

the time it was impossible to fly. Strong winds, heavy rain and even snow and ice made flying conditions extremely

hazardous.

5

But finally the first of three practical exams arrived – the navigation test. The examiner sets you a course that you

have to plan according to the weather, and then fly with him sitting beside you.

6

I passed this test, but I don’t know how. The second test involves flying cross-country to two other airports, which

you can choose, and landing at both. The important thing is to give the right messages to the air-traffic control people

and understand their replies.

7

After this alarming episode, the exercises in the flight-handling test were simple. As we complete the sixth exercise,

the examiner suddenly turned to me and said, ‘Congratulations – you’ve passed!’

8

I wasn’t sure why, because we usually land as slowly as possible. Then I turned round and realised straight away: we

were being followed by a British Airways jumbo jet!

A. A week which we had set aside for finishing the course came and went with no possibility of getting in the air at

all. And besides the problems with the weather, my second son was born, and that made it even more difficult to find

the time for lessons and studying.

9

B. But the real reason I chose this club was that a friend of mine, Andrew Wilkins, is the chief instructor there. He

impressed me by taking me out for a free flight just so that I could see what it was like.

C. Unfortunately, I got myself lost this time and flew too far east. I completely missed the first airport. However, I

flew over a car factory I recognised and managed to get back on course.

D. Along the way, he’ll take the controls and fly off course, just to get you lost. Then he’ll hand back the control to

you and expect you to find your way home.

E. One day I was asked by an air-traffic controller if I could see another aircraft ahead. I said yes, and immediately it

disappeared into a cloud. I just didn’t know what to do.

F. At the time, taking private lessons to learn how to fly was financially beyond me. So I had to delay my plans to

become a pilot for quite a while. It was twenty years, in fact, before I finally enrolled at a flying club in Hertfordshire.

G. Since getting my pilot’s license, I’ve been out flying a few times. The highlight so far was flying up to

Birmingham International Airport for a motor show with Andrew beside me. As we approached the way, the air-

traffic controller came on the radio asking for as much speed as our little plane could manage.

H. For months, my head was always in a book and my head hurt from all the facts, figures and flying instructions.

I. This feeling of needing a change coincided with my 40 th birthday, which started me thinking about what I’d been

doing all those years. When I left school all I had really wanted to do was fly.

SECTION 4: WRITING (6 points)

PART 1: Finish each of the following sentences in such a way that it means exactly the same as the sentences

printed before it.

1. Return the product to the shop if you have any complaints about it.

Should ___________________________________________________________________

2. It’s almost nine months since I stopped subscribing to that magazine.

I cancelled ________________________________________________________________

3. Her success went beyond her expectation.

Never ____________________________________________________________________

4. His fondness for the game increased with his proficiency.

The more _________________________________________________________________

5. They will not announce the findings until next week.

No announcement __________________________________________________________

6. Both of the lifts were out of order.

10

Neither ___________________________________________________________________

7. Simon hadn’t expected that he would feel so weak after the operation.

The operation left __________________________________________________________

PART 2: Finish each of the following sentences so that it has a similar meaning to the sentences printed before it,

using the word given. DON’T CHANGE THE WORD GIVEN. You must use between three and five words,

including the word given. There is an example at the beginning (0).

0. You must do exactly what the manager tells you. (CARRY)

You must carry out the manager’s instructions exactly.

1. I would like to be able to speak French. (HAD)

I wish ________________________________________________________ speak French.

2. It was raining cats and dogs. (TORRENTS)

The rain was ______________________________________________________________

3. The two theories appear to be completely different. (COMMON)

The two theories____________________________________________________________

4. It was wrong of you to borrow my book without asking. (HAVE)

You ____________________________________________before you borrowed my book.

5. When I was younger, this record was one of my favourites. (FAVOURITE)

This record used ______________________________________ mine when I was younger.

6. My sister finds commuting every day annoying. (PUT)

It’s difficult for my sister ___________________________________________ every day.

7. The police arrived as the thieves were committing the crime. (RED-HANDED)

The police ________________________________________________________________

8. The Mediterranean is warm, whereas the North Sea is much colder. (NOTHING)

The North Sea is ____________________________________________ the Mediterranean.

PART 3: In class, you have been discussing the statement ‘Old people should be looked after by members of their

family or it is better for them to live in nursing-home’. Your teacher has asked you to write an essay expressing

your own opinion.

Write your essay in 250-300 words.

……………………………………………………………………………………………………………………………

……………………………………………………………………………

……………………………………………………………………………………………………………………………

……………………………………………………………………………………………………………………………

……………………………………………………………………………………………………………………………

……………………………………………………………………………………………………………………………

……………………………………………………………………………………………………………………………

……………………………………………………………………………………………………………………………

11

SỞ GD&ĐT VĨNH PHÚC KỲ THI CHỌN HSG LỚP 10 THPT NĂM HỌC 2011-2012

Thời gian thi: 180 phút, không kể thời gian giao đề

PART II. GRAMMAR AND VOCABULARY

I. Complete the following sentences by choosing the correct answer among four options (A, B, C or D).

1. He's really shy _______ girl.

A. by B. at C. for D. with

2. The teacher _______ her to improve her drawing.

A. insisted B. encouraged C. made D. persisted

3. I couldn't quite ______ what they were doing because they were so far away.

A. bear out B. make out C. think out D. try out

4. The meal Mary cooked tastes_______.

A. well B. nice C. good D. worse

5. ______ at the party, we saw Ruth standing alone.

A. Arriving B. We arrived C. Arrived D. We were arriving

6. The people who______ the survey said that they had examined over 1,000 accidents.

A. gave B. proceed C. set D. conducted

7. The judge found him ______ of stealing and sent him to prison.

A. evil B. innocent C. guilty D. wicked

8. The house we have rented is______. So we will have to buy some beds, chairs, tables, etc.

A. unrestored B. unrepaired C. unfurnished D. undecorated

9. He was turned down for the job because he is ________.

A. qualified B. qualifying C. unqualified D. qualification

10. The trouble started only______ the other man came into the room.

A. when B. until C. and then D. too soon

11. _______, the disaster would not have happened.

A. Had you have obeyed the orders B. You had obeyed the orders

C. You obeyed the orders D. Had you obeyed the orders

12. _______ had booked in advance were allowed in.

A. Only who B. Only those who

C. Only who were those D. Only were those who

13. Traveling alone to a jungle is adventurous, ________.

A. if not impossible B. if it not impossible

C. when not impossible D. when it not impossible

14. I ______ the hot weather in the south.

A. use to B. used to C. am use to D. am used to

12

15. The meat looked very _______ to the dog.

A. invited B. invite C. inviting D. invitingly

II. Use the correct form of each word on the right to complete the numbered spaces provided in the passage.

Write your answers on your answer sheet.

The mysteries of the skies

Three hundred and fifty years before the first men looked down on the amazingly

beautiful surface of the moon from close quarters, Galileo’s newly built telescope (1)

_____________ him to look at the edge of the hitherto mysterious sphere. He saw that the

apparently (2) _____________ surface was not divinely smooth and round, but bumpy and

imperfect. He realized that although the moon might appear (3) _____________,

resembling a still life painted by the hand of a cosmic (4) ____________, it was a real

world, perhaps not very different from our own. This amounted to a great (5)

_____________ hardly to be expected in his day and age, although nowadays his (6)

_____________ may appear to some to be trivial and (7) _____________.

Not long after Galileo lunar’s observations, the skies which had previously been so (8)

_____________ revealed more of their extraordinary mysteries. Casting around for further

wonders, Galileo focused his lens on the (9) _____________ planet of Jupiter. Nestling

next to it, he saw four little points of light circling the distant planet. Our moon it appeared,

perhaps (10) ____________ in the eyes of those fearful of what the discovery might mean,

was not alone!

1. ABLE

2. LIVE

3. ACT

4. ART

5. ACHIEVE

6. CONCLUDE

7. SIGNIFY

8. ELUDE

9. STRIKE

10. FORTUNE

III. In the following passage, some numbered lines contain a word that shouldn’t be there. Tick (√) the

sentences that are correct and write the words that shouldn’t be there in the numbered space.

KEEPING YOUR DISTANCE

Personal space is a term that refers to the distance we like to keep between

ourselves and other people. When someone we do not know well gets too close that

we usually begin to feel uncomfortable. If such a business colleague comes closer

than 1.2 meters, the most common response is to move away. Some interesting

studies have been done in libraries. If strangers will come too close, many people get

up and leave the building, others use to different methods such as turning their back

on the intruder. Living in cities has made people to develop new skills for dealing

with situations where they are very close to strangers. Most people on so crowded

trains try not to look at strangers; they avoid skin contract, and apologize if hands

touch by a mistake. People use newspapers as a barrier between themselves and

other people, and if they do not have one, they stare into the distance, making sure

0 ___√___

00 someone

1________

2 ________

3 ________

4 ________

5 ________

6 ________

7 ________

8 ________

9 ________

13

they are not looking into anyone’s eyes. 10 _______

PART III. READING

I. Complete the following passage by choosing A, B, C or D to fill in each blank.

In recent years, there has been a remarkable increase into happiness. The researchers have come up a number

of factors which contribute to a definition of happiness.

First of all, there is, in some people, a moderate genetic predisposition to be happy, in other words, happiness

(1)_______ in families. And happiness seems to correlate quite strongly with the main dimensions of personalities:

extroverts are generally happier, neurotics are less so.

Second, people often report good social relations as a reason for their happiness. In particular, friends are a

great (2) ______ of joy, partly because of the agreeable things they do together, partly because of the way friends use

positive non-verbal (3) ______ such as caressing and touching, to affirm their friendship. Marriage and similar (4)

______ relationships can also form the basis of lasting happiness.

Third, job satisfaction undoubtedly (5) ______ overall satisfaction, and vice versa - perhaps this is why some

people are happy in boring jobs: it (6) ______ both ways. Job satisfaction is caused not only by the essential nature of

the work, but (7)_____ by social interactions with co-workers. Unemployment, on the contrary, can be a serious cause

of unhappiness.

Fourth, leisure is important because it is more under individual (8) ______ than most other causes of

happiness. Activities (9) _____ sport and music, and participation in voluntary work and social clubs of various kinds,

can give great joy. This is partly because of the (10) ______themselves, but also because of the social support of other

group members – it is very strong in the case of religious groups.

1. A. runs B. arrives C. goes D. descends

2. A. source B. origin C. base D. meaning

3. A. movements B. signals C. slogans D. motions

4. A. near B. tight C. close D. heavy

5. A. consists of B. applies to C. counts on D. contributes to

6. A. works B. effects C. makes D. turns

7. A. too B. as well C. also D. plus

8. A. check B. power C. choice D. control

9. A. so B. such C. like D. thus

10. A. facilities B. activities C. exercises D. amenities

II. Read the passage carefully, then choose the correct option (marked A, B, C or D) to answer the questions.

Scientists have established that influenza viruses taken from man can cause disease in animals. In addition,

man can catch the disease from animals. In fact, a greater numbers of wild birds seem to carry the virus without

showing any evidences of illness. Some scientists conclude that a large family of influenza virus may have evolved in

the bird kingdom, a group that has been on earth 100 million years and is able to carry the virus without contracting

14

the disease. There is even convincing evidence to show that virus strain are transmitted from place to place and from

continent to continent by migrating birds.

It is known that two influenza viruses can recombine when both are present in an animal at the same time.

The result of such recombination is a great variety of strains containing different H and N spikes. This raises the

possibility that a human influenza virus can recombine with an influenza virus from a lower animal to produce an

entirely new spike. Research is underway to determine if that is the way major new strains come into being. Another

possibility is that two animal influenza strains may recombine in a pig, for example, to produce a new strain which is

transmitted to man.

1. According to the passage, scientists have discovered that influenza viruses ______.

A. cause ill health in wild animals B. do not always cause symptoms in birds

C. are rarely present in wild birds D. change when transmitted from animals to man

2. What is known about the influenza virus?

A. It was first found in a group of very old birds.B. All the different strains can be found in wild birds.

C. It existed over 100 million years ago. D. It can survive in many different places.

3. According to the passage, a great variety of influenza strains can appear when______.

A. H and N spikes are produced B. animal and bird viruses are combined

C. dissimilar types of viruses recombine D. two viruses of the same type are contracted

4. New strains of viruses are transmitted to man by_______.

A. a type of wild pig B. diseased lower animals

C. a group of migrating birds D. a variety of means

5. It can be inferred from the passage that all of the following are ways of producing new strains of influenza

EXCEPT_______.

A. two influenza viruses in the same animal recombining

B. animal viruses recombining with human viruses

C. two animal viruses recombining

D. two animal viruses recombining in a human

III. Read the passage and choose the best answer from the four options marked A, B, C or D in the following

questions. Identify your answer by writing the corresponding letter A, B, C or D on your answer sheet.

Several hundred million years ago, plants similar to modern ferns covered vast stretches of the land. Some

were as large as trees, with giant fronds bunched at the top of trunks as straight as pillars. Others were the size of

bushes and formed thickets of undergrowth. Still others lived in the shade of giant club mosses and horsetails along

the edges of swampy lagoons where giant amphibians swam.

A great number of these plants were true ferns, reproducing themselves without fruits or seeds. Others had

only the appearance of ferns. Their leaves had organs of sexual reproduction and produced seeds. Although their

“flowers” did not have corollas, these false ferns (today completely extinct) ushered in the era of flowering plants.

Traces of these floras of the earliest times have been preserved in the form of fossils. Such traces are most commonly

found in shale and sandstone rocks wedged between coal beds.

15

Today only tropical forests bear living proof of the ancient greatness of ferns. The species that grow there are

no longer those of the Carboniferous period, but their variety and vast numbers, and the great size of some, remind us

of the time when ferns ruled the plant kingdom.

1. What does the passage mainly discuss?

A. Plant reproduction B. How to locate fossils

C. An ancient form of plant life D. Tropical plant life

2. The word “Others” refers to _________.

A. plants B. pillars C. trees D. fronds

3. Which of the following is NOT mentioned as a characteristic of the plants described in the passage?

A. They once spread over large areas of land.

B. They varied greatly in size.

C. They coexisted with amphibians, mosses, and horsetails.

D. They clung to tree trunks and bushes for support.

4. The word “true” is closest in meaning to which of the following?

A. accurate B. genuine C. straight D. dependable

5. The author states that fossils of early plant life are usually found in rocks located between deposits of _______.

A. coal B. shale C. sandstone D. corollas

IV. Read the passage carefully then fill in the blank a suitable word.

As swimming became a popular recreation in England during the 1860s and 1870s, several (1) ______ sports

developed, roughly patterned after land sports. (2) ______ them were water football (or soccer), water rugby, water

handball, and water polo, in which players rode on floating barrels, painted to look (3) ______ horses, and struck the

ball with a stick.

Water rugby became most popular of these sports, but somehow the water polo name became attached to it,

and it's been attached (4) ______ since.

As played in England, the object of the sport was for a player to touch the ball, with both (5) ______, at the

goal end of the pool. The goaltender stood on the pool deck, ready to dive on any opponent who was about to score.

Water polo quickly became a very rough sport, filled (6) ______ underwater fights away from the ball, and it

wasn't unusual for players to pass out for lack of air.

In 1877, the sport was tamed in Scotland by the addiction of goalposts. The Scots also replaced (7) ______

original small, hard rubber ball with a soccer ball and adopted (8) ______ that prohibited taking the ball under the

surface or, "tackling" a player unless he had the ball.

The Scottish game, which emphasized swimming speed, passing, and (9) ______ work, spread to England

during the early 1880s, to Hungary in 1889, to Austria and Germany in 1894, to France in 1895, and (10) ______

Belgium in 1900.

Water polo was the first team sport added to the Olympic program, in 1900.

PART IV: WRITING

I. Write the new sentences using the given word. Do not change the word given in any way.

16

1. They have discovered some interesting new information. (LIGHT)

2. They suspended Jack for the next two matches. (BANNED)

3. I really want to see her again. (DYING)

4. She was so beautiful that I couldn't stop looking at her. (EYES)

5. We are looking forward to watching the program. (WAIT)

II. Rewrite each of the following sentences so that it has a similar meaning to the original one.

1. If you changed your mind, you would be welcomed to join our class.

→ Were you______________________________________________________

2. I'd rather not go out this afternoon.

→ I do not feel____________________________________________________

3. Adeles tries hard, but she doesn't get anywhere.

→ However______________________________________________________

4. It is thought that the boss is considering raising wages.

→ The boss______________________________________________________

5. His disabilities did not prevent him from sailing around the world.

→ Despite the fact_________________________________________________

6. I didn't arrive in time to see her.

→ I wasn't_______________________________________________________

7. I'd prefer you not to smoke.

→ I'd rather______________________________________________________

8. The mother smiled happily. She took the baby in her arms.

→ Smiling________________________________________________________

9. The noise next door did not stop until after midnight.

→ It was not ______________________________________________________

10. You can ring this number whenever there is any difficulty.

→ Should _________________________________________________________

SỞ GIÁO DỤC VÀ ĐÀO TẠO KỲ THI CHỌN HỌC SINH GIỎI LỚP 12 - VÒNG 1

LONG AN Ngày thi: 06/10/2011

Thời gian: 180 phút (Phần A & B)

A. READING & WRITING : (80 marks)

I. Read the passage and answer the questions that follow. (15 marks)

Steve grew up in a children’s home in Scotland. Every week, the staff took a proportion of the children’s pocket

money to pay for holidays. Little Steve developed a daring strategy to hang on to what little money he had. He

laughs about it now. “I used to swallow all my coins. That got them really furious, so they’d put me in a room on

my own as a punishment. After a few minutes in there, I’d hit myself on the chest and cough the money back up.”

17

Đề chính thức Bảng A

Since then, Steve has turned into Steve Starr, a professional regurgitator who does up to four shows a day, and

can demand fees of $500-$2,000 a show.

Everything Steve swallows comes back dry, except for the goldfish. They swim about in his stomach in the

water that he swallows for them first. After ten minutes they resume their normal lives in a goldfish bowl. “They

never die”, says Steve.

Medical experts might have a few worries about Steve. The sight and sound of him swallowing and bringing

back a snooker ball sometimes causes even normally calm people to panic. He also smokes a cigarette, retains the

smoke in his stomach, then swallows some butane gas and mixes the two. Next he swallows some washing-up

liquid, blows a huge bubble, brings up the smoke and gas inside the bubble, cuts the bubble off, gets someone to

set light to it, and bang!

Physically, Steve doesn’t believe he is any different from the rest of the human race, who use their stomachs

simply to digest food. “It’s all done by muscle control”, he says. “I imagine a little pair of hands in there doing

everything, controlled by my brain. I’m sure I could teach anyone to do what I do”.

1. Why was Steve punished in the children’s home?

2. What does the sentence “Steve grew up in a children’s home” tell you about Steve’s early childhood?

3. What does the sentence “They’d put me in a room on my own as a punishment” tell you about the children’s

home?

4. Why does the bubble go bang when someone sets light to it?

5. What, according to Steve, is the secret of his skill?

II. Read the following passage and then choose the best answer (A, B, C or D) to complete each of the numbered

blanks. (15 marks)

THE LANGUAGE OF TEARS

The ability to weep is a uniquely human form of emotional response. Some scientists have suggested that

human tears are evidence of an aquatic past – but this does not seem very likely. We cry from the moment we

enter this (1) _________ for a number of reasons. Helpless babies cry to (2) _________ their parents that they are

ill, hungry or uncomfortable. As they (3) _________ they will also cry just to attract parental attention and will

often stop when they get it.

The idea that (4) _________ a good cry can do you (5) _________ is a very old one and now it has scientific

(6) _________ since recent research into tears has shown that they (7) _________ a natural painkiller called

enkaphalin. By (8) _________ sorrow and pain this chemical helps you to feel better. Weeping can increase the

quantities of enkaphalin you (9) _________.

Unfortunately, in our society we impose restrictions upon this naturally (10) _________ activity. Because

some people still regard it as a (11) _________ of weakness in men, boys in particular are admonished when they

cry. This kind of repression can only increase stress, both emotionally and physically.

Tears of emotion also help the body (12) _________ itself of toxic chemical (13) _________, for there is

more protein in them than in tears resulting from cold winds or other irritants. Crying comforts, calms and can be

18

very enjoyable – (14) _________ the popularity of the highly emotional films which are commonly (15)

_________ “weepies”. It seems that people enjoy crying together almost as much as laughing together.

1. A. world B. place C. earth D. space

2. A. communicate B. persuade C. inform D. demonstrate

3. A. evolve B. change C. develop D. alter

4. A. doing B. making C. getting D. having

5. A. better B. fine C. good D. well

6. A. validity B. truth C. reality D. reason

7. A. contain B. retain C. hold D. keep

8. A. struggling B. fighting C. opposing D. striking

9. A. construct B. achieve C. provide D. produce

10. A. curing B. treating C. healing D. improving

11. A. hint B. symbol C. feature D. sign

12. A. release B. rid C. loosen D. expel

13. A. rubbish B. waste C. leftovers D. remains

14. A. consider B. remark C. distinguish D. regard

15. A. named B. entitled C. subtitled D. called

III. Read the following text and then choose the best phrase or sentence, given below, to fill each of the gaps. Write

one letter (A-P) in each of the numbered gaps. Each phrase is only used once. Some of the suggested answers do

not fit at all. (15 marks)

Archaeologists in Iraq have discovered the world’s oldest “statue” – a stone, standing four feet high, covered

with plaster (1) __________ of a human being.

This “stone man” dating from 11,000 years ago, (2) __________ who were emerging from the pre-agricultural

Stone Age into the Neolithic world of early farming.

The statue, probably of religious significance, was located inside a prehistoric house – one of the earliest

sophisticated buildings (3) __________. Investigations have shown that the house had (4) __________ with clay-

coated, lime-plastered walls and floor.

So far excavation have unearthed three buildings containing seven standing stones, (5) __________ retain traces

of lime plaster which once covered them.

However, (6) __________ shows evidence of having been shaped into the likeness of a human being. It is 30

centuries older (7) __________ previously known oldest statue. (8) __________that each building appears to have

had at least one standing stone inside it, and that one house actually had three.

The plaster-covered human shaped obelisk (9) __________ shoulders and the stumps of arms and part of a

neck. The “head”, however, (10) __________.

A. ever discovered by archaeologists

B. molded into the shape

C. to have been built

19

D. strangely carved

E. was fashioned by people

F. excavations have revealed

G. only one of these

H. neither of them

I. beautifully finished

J. than the remaining

K. has what appear to be

L. seems to have broken off

M. four of which

N. has been missing

O. it has been decided

P. than the world’s

IV. Read the text below and think of the word which best fits each space. Use only one word in each space. (20

marks)

The worry about salt is that it may (1)__________ high blood pressure. Chemically, salt (2) __________of

sodium and chloride ions, both of (3)__________are common in the human (4)__________and are important for

many physiological and biochemical (5) __________. We not only need salt, we eat salt, but too (6) __________

may still be bad for us. Although the idea of a (7)__________between salt and high blood pressure (8)

__________back to 2000 BC, there is still no scientific (9)__________as to whether this so or not. One reason for

this (10) __________to agree is that individual salt intake (11) __________enormously from day to day, and so

reliable measures of intake are hard to come (12) __________.

Those who believe that salt does (13)__________to high blood pressure (14)__________to the high incidence of

high blood pressure in countries that eat a very (15) __________diet. In Japan, for instance, where salted fish is

an important part of the diet, high blood pressure and (16)__________ complications are common, (17)

__________ among some Amazonian and African tribes, which have a low intake of salt, they are almost (18)

__________.

But (19)__________there is this neat relation between salt intake and the incidence of high blood pressure

between countries, it doesn’t seem to apply (20) __________those countries themselves. Studies, for instance, of

couples who have a similar salt intake don’t show any consistency in how often they develop high blood pressure.

V. There are ten mistakes in the following passage. Find and correct them. Follow the example.

(15 marks) Example: Line 1: 0. which when

FAMILY HISTORY

In an age which technology is developed faster than ever before, many people are being attracted by the idea

of looking back into the past. One way they can make this is by investigating their own family history. They can

try to find out more about what their families came from and what they did. This is now a fast-growing hobby,

especially in countries with a fairly short history, alike Australia and the United States.

20

It is one thing to spend some time going through a book on family history and to take the decision to

investigate your own family’s past. It is quite another to carry out the research work successfully. It is easy to set

about it in a disorganizing way and cause yourself many problems that could have avoided with a little forward

planning.

If your own family stories say you that you are connected with a famous character, whether hero or criminal,

not to let this idea take over your research. Just treat it as an interesting possibility. A simple system for collecting

and storing your information will be adequate to start with; a more complex one may only get in your way. The

most important thing, though, is to get starting. Who knows what you might find?

SỞ GIÁO DỤC VÀ ĐÀO TẠO KỲ THI CẤP TỈNH CHỌN HỌC SINH GIỎI LỚP 12 – VÒNG 1

LONG AN Ngày thi : 06/10/2011

MÔN THI : TIẾNG ANH BẢNG B

ĐỀ CHÍNH THỨC Thời gian làm bài : 180 phút cho cả hai phần A và B

B/. READING AND WRITING (80 points)

I/. Read the passage carefully and answer the questions. Write your answers on your paper.

(20 points)

One day more than 200 years ago a man complained to his doctor that he had a ball in his throat. “It goes up

and down when I breathe”, he said. The doctor laughed at him and said, “There is nothing in your throat”.

Soon after the man died and the doctor discovered that there was a steel ball in his throat. This unfortunate

man worked in needles factory and in the process of grinding steel to make the needles, he had breathed in particles of

it which soon accumulated and formed a lump in his throat. Ever since that time workers in factories producing

needles have had to wear masks to protect themselves.

Needles were first used by the Stone Age man about 50.000 years ago to stitch together pieces of pelt to make

clothes. At the time primitive men used the bones of animals and fish, and even thorns, to make ‘needles’- rough

stone tools with a hole in one end. Later the Romans and the British used iron and bronze for making needles.

Today’s needles are made from steel wire. When you examine a needle it looks quite a simple object but there

are fifteen different stages a steel wire has to go through before it becomes a needle. Let us look at some of them.

First the worker cuts a steel wire into the length of two needles. Then he straightens out the wire and sharpens

it at both ends. At the center he pierces two eyes: this forms two needles joined end to end. Usually machines are used

to break up the needles; only very good quality needles are broken by hand.

The needles are by no means ready yet. They must go into a special machine which removes the rough edges

from around the eye of each needle. After this the worker puts all of them into a small furnace called a ‘muffle’, and

when the needles are red hot he plunges them into a container of cold oil. This process is repeated many times to

harden the needles and give them elasticity. Washing, rolling and polishing follow, after which needles are put into

packets for sale.

21

You can see needles everywhere today, in your home and in the hospital where they are used as surgical

instruments. Compasses and record-players too require different types of needles. The Stone Age man would indeed

be amazed if he could see how many uses we have made of his simple invention.

1.Why did the doctor laugh at the man who went to him with a complaint?

2.Was the doctor right in laughing at the man? Why?

3.How had the steel ball formed inside the man’s throat?

4.Why do workers in needles factories wear masks?

5.Why are the needles heated and then cooled many times?

II/.Use the words below to fill in the blanks. Each correct word is only used once. Write the words in the

corresponding numbered places on your paper. (30 points)

Passage 1 (10 points)

well – which – works – also – after – grew – order – from – where – as

Leonardo da Vinci is perhaps most famous for his (1) . . . . . . . . of art. Among his more well-known paintings

are the “Mona Lisa” and “The Last Supper” Leonardo was an accomplished painter but he was (2) . . . . . . . . . .an

architect, a sculptor, an engineer, a scientist and a musician. He was one of the most talented men of all time.

Leonardo was named (3) . . . . . . . . . .the town of Vinci (4) . . . . . . . . he was born. Leonardo da Vinci actually

means “Leonardo of Vinci”. He (5) . . . . . . . . . . . . . . . up in the busy city of Florence which was then a center of

education and of the arts. Leonardo did not receive much formal education. He became an apprentice to a painter and

sculptor. Verocchio, (6) . . . . . . . . . . . . . . . whom he learned his craft. He conversed with the scholars and noblemen

who came to order works of art and learned much from them.

Leonardo was a very curious and creative person. He believed that (7) . . . . . . . . . an artist, he had to acquire

an understanding of objects in (8) . . . . . . . . . . to paint them well. Before he embarked on a new painting, he would

find out all the information he could gather about the subjects he was painting. He would also draw sketches of these

subjects in a notebook.

When he was alive, he was accepted as a very learned man. The breath of his knowledge can be seen in more

than thirty of his notebooks (9) . . . . . . . . . . . . . . .are still intact today. The notes and drawings in the notebooks

included plans for a tank, a helicopter as (10) . . . . . . . . . . . . . . . as a detailed study of the human anatomy.

Passage 2 (20 points)

ENGLISH UNIVERSITIES AND COLLEGES

consists – just – others – some – proportion – lasts – both – departments – practical – number – changes-spends

All English universities except Oxford and Cambridge are fairy new. London University is the biggest of the

modern English Universities and (1) . . . . . . . . . . . . . . of a great variety of colleges and other institutions including

medical schools.

22

A university usually has (2) . . . . . . . . . . . . . . . faculties and departments. The most common faculties are

medicine, law, arts, science, and theology. The (3) . . . . . . . . . . . . . . include engineering, economics, commerce,

agriculture, music and technology. After taking examinations a university graduate is awarded with the Degree of

Bachelor of Arts or Science, or Engineering, or Medicine, etc. depending on their field of study.

Each faculty is headed by one or more professors who are helped by a staff of teachers called lecturers. Professors

and lecturers spend (4) . . . . . . . . .. . . . . . . . of their time giving lectures to a large (5) . . . . . . . . . . . . . . . . of

students or studying with smaller groups and here the students have a chance to argue and discuss.

All universities admit men and women, but the (6) . . . . . . . . . . . . . . . . of men is rather more than 75%. Most of

the universities provide hostel accommodation for their students. At a university the course of studies (7) . . . . . . . . . .

. . . . . . six years and the curriculum is wide.

There are many special types of colleges in Great Britain too. They give a specialized training. They are medical,

teachers’, technical colleges and many (8) . . . . . . . . . . . .. . . One can see colleges within universities. The course of

studies at a college is (9) . . . . . . . . . . . . . . three years. At medical colleges the students study various subjects, learn

to treat patients and have (10) . . . . . . . . . . . . . . . . work at hospitals. After graduating from the college they are given

a certificate.

III/. Each of the following sentences has 4 underlined parts, marked A,B,C and D. Choose the underlined part

of the sentence that needs corrections and then give the correct word. Write on your paper (30 points)

Ex :

0. They have (A)been (B) living here (C) since six years (D) now.

Answer : 0 : (C), for

1. Each of the (A) s t u d e n ts (B) in t h e accounting class (C) ha s t o type (D) t he ir own research paper this

semester.

2. (A) Plants and animals (B) live in the see sank (C) to the sea bed (D) when they died.

3. How (A) m u c h times (B) d id Rich a n d J e nn i f e r h a v e to do the experiment before they

(C) ob t a i ne d t h e r e s u lts they had (D) b ee n e x pe cti n g .

4. Mrs. Stevens, along (A) w ith , (B) he r c ou s i n s f ro m New Mexico, (C) a r e planning (D) to a tt e n d the

festivities.

5. (A) J oe l giving up (B) s m o ki n g ha s (C) c au s e d h im to g a in weight and (D) b e c o m e i rr it ab le with his

acquaintances.

6. I (A) will send this book (B) to you (C) before I (D) will finish it.

7. (A) Establishing in 1984 (B) for students who wanted to study art and music subjects, LaGuardia (C) was the

first public school of (D) its kind.

8. Alloys of gold and copper (A) have been (B) widely (C) using in (D) various types of coins.

9. We should put (A) people first and (B) save people’s (C) lives is still the top priority of our (D) relief work.

10.Scientists and economists (A) believe that human beings (B) can never (C) use away all the (D) mineral

resources on Earth.

23

Page 3/4

SỞ GD&ĐT KỲ THI CHỌN HỌC SINH GIỎI CẤP TỈNH LỚP 11 THPT

QUẢNG BÌNH Thời gian làm bài: 180 phút (không kể thời gian giao đề)

III. LEXICO-GRAMMAR

Part 1. Choose the word or phrase (A, B, C or D) that best completes each sentence. (10 pts)

21. Does anyone know ............. that jacket might be?

A. who B. whom C. whose D. which

22. The cheetah runs 70 miles ............. hour. It’s the fastest animal in the world.

A. each B. per C. a D. one

23. Joe seemed to be in a good mood, ......... he snapped at me angrily when I asked him to join us.

A. yet B. so C. for D. and

24. There are several means of mass communication. The newspaper is one. Television is .............

A. the another B. the other C. other D. another

25. I read in one paper that they are ............. married.

A. getting B. got C. being D. having

26. There was hardly ............. food left in the fridge.

A. more B. any C. no D. some

27. We spent the ............. days on the beach.

A. few last sunny B. last sunny few C. last few sunny D. few sunny last

28. ............. not fallen over, the athlete would have won the gold medal.

A. If he B. If not C. Unless he had D. Had he

29. Rarely............. to work on his own.

A. he is seen B. does he seen C. is he seen D. does he

30. “We’re going to the seaside”. - “Can............. ?”

A. I come as well B. also I come C. I too come D. I as well come

Part 2. Complete each sentence with a suitable form of one of the phrasal verbs in the box. Use each one once

only. (5 pts)

31. Why do prices keep ............. ............. ?

32. This new game is really............. ............. . Everyone is playing it.

33. It’s time your brother ............. ............. and started a family.

34. When she retired, she ............. ............. gardening so she wouldn’t get bored.

35. I’m sorry to ............. you ............. but I can’t take you to the airport after all.

Part 3. Complete the following sentences with appropriate form of the word in block capitals. (5 pts)

36. ACT There is a saying in English: “ ............. speak louder than words.”

37. QUALIFY He won the discus event at the Olympic Games but was later ............. when a medical

check proved that he had been taking drugs.

24

catch on go up let down take up settle down

38. NATION Do you have a ............. costume in your country?

39. LAND The plane had to make a crash ............. in a field.

40. TROULE Traveling in big cities is becoming more ............. everyday.

Part 4. Put the verbs given in the brackets into the appropriate tenses or forms. (10 pts)

41. “You were late for your dental appointment.”

“I know. I (not/ stay) .................. so long at the library.”

42. It is necessary that everyone (be) .................. calm in times of danger.

43. If you want to see us, come to Tom’s on Sunday. We (wait) .................. for you there at

midday.

44. It was a fine day and the roads were crowded because a lot of people (rush) ..................

to the seaside.

45. Those were the highest words of praise they ever (hear) .................. from the old man.

46. Our little children enjoy (take) .................. to the water park every Sunday.

47. I don’t feel good. I (stay) .................. home from work tomorrow.

48. It won’t be safe to use these stairs until they (repair) .................. .

49. The city is now crowded with the people who (seek) .................. employment.

50. The old man is said (leave) .................... all his money to an old people’s home when he

died.

Part 5. In each of the following sentences, the 4 words or phrases are marked A, B, C or D. Identify the one

underlined expression that is not correct and correct it. (5 pts)

51. Almost American Indian cultures have been agricultural societies since 2000 BC.

A B C D

52. Would you like to contribute for our earthquake fund?

A B C D

53. The violence is a very great problem in the world.

A B C D

54. Now that the stress of examinations are over, we can go somewhere for our holiday.

A B C D

55. Passengers are advised not to leave their luggage attended.

A B C D

IV. READING COMPREHENSION

Part 1. Read the passage and choose the best answer (A, B, C or D) which best fits each space. (10 pts)

In the past people suffered (56) .................... a disease called scurvy. Their gums bled, their skin became

rough, their wounds did not (57) .................. and their muscles wasted away. The (58) ................... of these

symptoms was a lack of vitamin C; people ate preserved meats and foods and could not get fresh vegetables and

fruits.

The best (59) ................... of vitamin C are oranges, lemons, grapefruit, cantaloupes, strawberries, and

fresh vegetables. These fruits must be fresh because vitamin C is destroyed by heat, storage, or exposure (60)

25

.................. air.

Although today more people (61) ................... vitamin C pills than any other supplement, some people still

have scurvy, (62) .................. some of the elderly, alcoholics, and the chronically ill.

Research shows that vitamin C reduces the (63) ................... of colds and can help prevent cancer. There

is also evidence that vitamin C prevents heart disease, (64) ................... wound healing, helps prevent gum

disease, and helps protect us from pollutants such as cigarette smoke. Some recent research also shows that

vitamin C has a positive effect on some mental (65) ................... and increases life span.

56. A. from B. with C. by D. at

57. A. close B. recover C. heal D. get well

58. A. reason B. cause C. origin D. signs

59. A. store B. resources C. provider D. sources

60. A. to B. towards C. in D. by

61. A. consume B. drink C. take D. have

62. A. containing B. including C. consisting D. like

63. A. severity B. seriousness C. gravity D. importance

64. A. hastens B. hurries up C. quickens D. speeds

65. A. chaos B. confusions C. disorders D. disturbances

Part 2. Read the text below and think of the word which best fits each space. Use only ONE word in each

space. (10 pts)

At sixteen Henry Vincent was separated from his family as a result of the war. He wandered aimlessly from

(66) ................ country to another (67) ................ finally setting down in Australia, where he trained (68) ................ an

electronics engineer. He established his own business but it called for so much work that marriage was out of the

(69) ................ .

His retirement suddenly (70) ................ him realize (71) ................ lonely he was and he decided to

(72) ................ up a hobby. With his interest in electronics, amateur radio seemed a natural choice. He installed his

own equipment and obtained a licence and his call sign, which is the set of letters and numbers used to announce

oneself when making radio contact with other radio amateurs all over the world.

Soon Henry had a great many contacts in far-off places. One in particular was a man in California with whom

he had much in common. One night the man in California (73) ................ to mention the village in Europe he had

come from. Suddenly, Henry realised that this man was in fact his younger brother, Peter. At first, the two brothers

were at a (74) ................ for words but then little by little they filled in the details of their past lives and not

(75) ................ afterwards Henry Vincent flew to California to be reunited with his brother.

Part 3. Read the following passage and choose the correct answers (A, B, C, or D). (5 pts)

May 7, 1840, was the birthday of one of the most famous Russian composers of the nineteenth century:

Peter Illich Tchaikovsky . The son of a mining inspector, Tchaikovsky studied music as a child and later studied

composition at the St. Petersburg Conservatory. His greatest period of productivity occurred between 1876 and

1890, during which time he enjoyed the patronage of Madame von Meck, a woman he never met, who

gave him a living stipend of about $1,000 a year . Madame von Meck later terminated her friendship with

26

Tchaikovsky, as well as his living allowance, when she herself was facing financial difficulties . It was during the

time of Madame von Meck's patronage, however, that Tchaikovsky created the music for which he is most

famous, including the music for the ballets of “Swan Lake” and “The Sleeping Beauty”. Tchaikovsky's

music, well known for its rich melody and sometimes melancholy passages, was one of the first that brought

serious dramatic music to dance . Before this, little attention had been given to the music behind the dance.

Tchaikovsky died on November 6, 1893, ostensibly of cholera, although there are now some scholars who argue

that he committed suicide.

76. With what topic is the passage primarily concerned?

A. The life and music of Tchaikovsky.

B. Development of Tchaikovsky's music for ballets.

C. Tchaikovsky's relationship with Madame von Meck.

D. The cause of Tchaikovsky's death.

77. Where is the best place in the passage to add the following sentence?

She had commissioned some works from him and had so admired his music that

she agreed to support him.

A. In line 2 after the word “Tchaikovsky” B. In line 6 after the word “year”

C. In line 7 after the word “difficulties” D. In line 12 after the word “dance”

78. In lines 5, the phrase “enjoyed the patronage of” probably means:

A. liked the company of B. was mentally attached to

C. solicited the advice of D. was financially dependent upon

79. According to the passage, all of the following describe Madame von Meck EXCEPT:

A. She had economic troubles.

B. She was generous.

C. She was never introduced to Tchaikovsky.

D. She enjoyed Tchaikovsky's music.

80. According to the passage, “Swan Lake” and “The Sleeping Beauty” are:

A. dances B. songs C. operas D. plays

V. WRITING

Part 1. Finish each of the following sentences in such a way that it is as similar as possible in meaning to the

sentence printed before it. (5 pts)

81. I am only interested in why he did it.

→ The only thing.............................................................................

82. House prices have risen dramatically this year.

→ There has been ...........................................................................

83. I can’t understand how this new computer works.

→ This new computer works ..........................................................

84. She didn’t inherit anything under her uncle’s will.

→ Her uncle didn’t .........................................................................

27

85. I only recognized him when he came into light.

→ Not until .....................................................................................

Part 2. Complete the second sentence so that it has a similar meaning to the first sentence, using the word

given. Do not change the word given. You must use between two and five words, including the word given.

86. People claim that he was the best tennis player of his times. said

→ He................................. the best tennis player of his times.

87. They left early because they didn’t want to get caught in the traffic. avoid

→ They left early in order ................................. in the traffic.

88. Why didn’t they tell me about these changes earlier? should

→ I ................................. about these changes earlier.

89. He had a very traditional upbringing, didn’t he? traditionally

→ He ................................., wasn’t he?

90. I would like to express my thanks for everything you have done for me. thankful

→ I’d like to say................................. am for everything you have done for me.

Part 3. Composition (10 pts)

The world is experiencing a dramatic increase in population. This is causing problems not only for poor,

developing countries, but also for industrialized and developed nations. Describe some of the problems that

overpopulation causes and suggest at least one possible solution. You should write at least 200 words.

..................................................................................................................................................................

..............................................................................................................................................................................

---------- THE END ----------

PART TWO. LEXICO – GRAMMAR (30p.)

Question 1. Choose the word or phrase which best completes each sentence. Write your answers (A, B, C, or D) in

the space provided under this part. (10p.)

1. The ______ of two houses prove such a financial burden that they were forced to sell one

A. upshot B. upkeep C. uproar D. upsurge

2. In his student days, he was as poor as a church ______

A. beggar B. miser C. mouse D. pauper

3. Harry doesn’t ______ to great fame and fortune, he just wants to make a decent living.

A. crave B. hanker C. yearn D. aspire

4. I wrote to them a fortnight ago but ______ I haven’t had a reply

A. as yet B. these days C. so long D. just now

5. I couldn’t stop myself from ______ with boredom during the lecture.

A. sighing B. gasping C. panting D. blowing

6. She didn’t show even a ______ of emotion when the court found her guilty.

A. gleam B. wink C. flicker D. flash

7. It’s not surprising that he became a writer because he always longed to see his name______.

28

A. in type B. in print C. in letters D. in edition

8. The police are looking into new ways of ______ major crime.

A. contending B. wrestling C. combating D. striving

9. The technological and economic changes of the 19th century had a marked ______ on workers.

A. cause B. effect C. impact D. consequence

10. The first sign of vitamin A disorder is night ______.

A. loss of sight B. lack of vision C. invisibility D. blindness

11. The ______ are against her winning a fourth consecutive gold medal.

A. chances B. bets C. prospects D. odds

12. References can have a considerable ______ on employment prospects.

A. cause B. decision C. weight D. bearing

13. The prospects of picking up any survivors are now ______.

A. thin B. narrow C. slim D. restricted

14. From time to time he ______ himself to a weekend in a five-star hotel.

A. craves B. indulges C. treats D. benefits

15. Men still expect their jobs to take ______.

A. superiority B. imposition C. priority D. seniority

16. The police have been ordered not to ______ if the students attack them.

A. combat B. rebuff C. retaliate D. challenge

17. Meg had a ______ escape when she was hang-gliding yesterday.

A. slender B. close C. near D. narrow

18. I can’t tell you the exact amount, but I can give you a ______ estimate.

A. smooth B. tidy C. rough D. similar

19. Marge walked away from the discussion. Otherwise, she ______ something she would regret later.

A. will say B. said

C. might say D. might have said

20. You are not supposed to park on the hard ______ except in an emergency.

A. lane B. shoulder C. leg D. area

Question 2. There are TEN mistakes in this paragraph. Write them down & give the correction. Write your

answers in the space provided. (5p.)

29

Line 1

2

3

4

5

6

7

8

9

10

11

12

Skiing is one of the most popular sports in the world. According to recent estimation, about one

hundred million of people ski regularly or occasionally.

Sliding across the snow on skis is also one of the most ancient methods of transport known to the

man. It has demonstrated that men were already traveling across the snow by means of primitive skis

before the invention of the wheel. In the Asiatic region of Altai and in Scandinavia, for example, the

remains of skis have been found which dated back to 4,000 BC. Further evidence is supplied by ancient

cave paintings which depict people skiing, and a Norwegian saga which tells the story of an invasion of

its territory 8,000 years ago by a tribe of skiers who came from the north.

Nowadays, skiing, apart from a sport, has become a big industry and a notable feature of leisure

culture. Ski resorts and all the activity that they generate is the main source of wealth in many mountain

regions, which were previously remote and inaccessible. And far from its once elitist image, skiing is

now enjoyed by an increasingly broader spectra of society.

Your answers

Line Mistake Correction

1.

2.

3.

4.

5.

6.

7.

8.

9.

10

Question 3. Complete each of the following sentences with the correct form of one verb from the box and a

suitable particle. Write your answers in the space provided (5p.)

Lag jump lie fall bring

Work bring jot get tick

1. My purse has just _______ behind the sofa even though I thought I’d lost it for good.

2. It was Tony who would always _______ the rest of the party. He would stop at every window and gaze for hours.

3. He’s a nature enthusiast and he will certainly _______ the opportunity of visiting the wildlife reserve in Ohio.

4. “Keep talking while I _______ my notes.” she said.

5. Unless your wife stops leading her extravagant lifestyle, you won’t _______ on the poor salary you obtain.

6. The exact cost of the whole venture isn’t known yet, but our best accounts have been trying to _______(it).

7. I don’t quite like the new apprentice’s approach. He seems to _______ the job.

30

8. Those naughty boys went on making terrible noise in the park even though they had been _______ by the annoyed

constable.

9. His heart attack was _______ by too much stress at work.

10. I am not friends with Peter anymore. We’ve_______.

Question 4. Use the correct form of each bracketed word in the numbered space. Write your answers in the space

provided. (5 p.)

The spiral and the helix

They are everywhere, (1-GRACE), curving shapes whose incredible (2- REGULAR) contrasts so sharply

with the random world around them. We call them spirals and helices but that hardly does (3-JUST) to their diversity

or their significance. Over the centuries, mathematicians have identified many different types, but the most intriguing

are those that (4-REPEAT) occur in the natural world.

The need to (5-RAVEL) the mysteries of the existence of spirals and helices has exercised some of the best

scientific brains in the world and opened the way to a number of (6-BREAK) in fields as widely varied as genetics

and meteorology.

The most (7-SPETACLE) spirals on earth are also the most unwelcome hurricanes. Their (8-AWE) power

comes from the sun’s heat, but they owe their shape to the force caused by the rotation of the earth. After (9-

NUMBER) years of study, however, Nature’s spirals and helices have yet to (10-CLOSE) all their secrets. For

example, why, astronomers wonder, are so many galaxies spiral-shaped?

Your answers

1. 6.

2. 7.

3. 8.

4. 9.

5. 10.

Question 5. Read the text below and decide which answer (A, B, C or D) best fits each gap. Write your answers in

the space provided. (10p.)

Although the earth was formed about 4,500 million years ago, human beings have inhabited it for less than half a

million years. Within this time, population has increased hugely and people have had a vast (1)...... upon the earth.

They have long been able to (2)...... the forces of nature to use. Now, with modern technology, they have the power to

alter the balance of life on earth.

Reports back from the first astronauts helped dispel the dangerous (3)...... that the world had no boundaries and had

limitless resources. Moreover, ecologists have shown that all forms of life on earth are interconnected, so it (4)......

that all human activity has an effect on the natural environment.

In recent years, people have been putting the environment under stress. As a result, certain (5)...... materials

such as timber, water and minerals are beginning to (6)...... short. Pollution and the (7)...... of waste are

31

already critical issues, and the (8)...... of the environment is fast becoming the most pressing problem (9)......

us all. The way we respond to the challenge will have a profound effect on the earth and its life support

(10)...... .

1. A. imprint B. indication C. impression D. impact

2. A. put B. make C. place D. stand

3. A. judgment B. notion C. reflection D. concept

4. A. results B. follows C. complies D. develops

5. A. raw B. coarse C. crude D. rough

6. A. turn B. come C. go D. run

7. A. disposal B. displacement C. dismissal D. disposition

8. A. state B. situation C. case D. circumstance

9. A. encountering B. opposing C. meeting D. confronting

10. A. projects B. systems C. methods D. routines

PART THREE: READING (30p.)

Question 1. Read the following passage and choose the best answer A, B, C or D for each question. Write your

answers in the space provided. (10p.)

We are descendents of the ice age. Periods of glaciation have spanned the whole of human existence for the

past 2 million years. The rapid melting of the continental glaciers at the end of the last ice age spurred one of the most

dramatic climate changes in the history of the planet. During this interglacial time, people were caught up in a

cataclysm of human accomplishment, including the development of agriculture and animal husbandry. Over the past

few thousand years, the Earth’s climate has been extraordinarily beneficial, and humans have prospered exceedingly

well under a benign atmosphere.

Ice ages have dramatically affected life on Earth almost from the very beginning. It is even possible that life

itself significantly changed the climate. All living organisms pull carbon dioxide out of the atmosphere and eventually

store it in sedimentary rocks within the Earth’s crust. If too much carbon dioxide is lost, too much heat escapes out

into the atmosphere. This can cause the Earth to cool enough for glacial ice to spread across the land.

In general the reduction of the level of carbon dioxide in the atmosphere has been equalized by the input of

carbon dioxide from such events as volcanic eruptions. Man, however, is upsetting the equation by burning fossil

fuels and destroying tropical rain forests, both of which release stored carbon dioxide. This energizes the greenhouse

effect and causes the Earth to warm. If the warming is significant enough, the polar ice caps eventually melt.

The polar ice caps drive the atmospheric and oceanic circulation systems. Should the ice caps melt, warm

tropical waters could circle the globe and make this a very warm, inhospitable planet.

Over the past century, the global sea level has apparently risen upwards of 6 inches, mainly because of the

melting of glacial ice. If present warming trends continue, the seas could rise as much as 6 feet by the next century.

This could flood coastal cities and fertile river deltas, where half the human population lives. Delicate wetlands,

where many marine species breed, also would be reclaimed by the sea. In addition, more frequent and severe storms

would batter coastal areas, adding to the disaster of the higher seas.32

The continued melting of the great ice sheets in polar regions could cause massive amounts of ice to crash

into the ocean. This would further raise the sea level and release more ice, which could more than double the area of

sea ice and increase correspondingly the amount of sunlight reflected back into space. The cycle would then be

complete as this could cause global temperatures to drop enough to initiate another ice age.

1. According to the passage, carbon dioxide is stored in each of the following EXCEPT

A. polar ice caps. B. sedimentary rocks. C. rain forests. D. fossil fuel.

2. What does the final paragraph of the passage mainly discuss?

A. The relationship between the ocean and the sun

B. The amount of sunlight reflected into space

C. A rise in global temperatures

D. The conditions that could lead to an ice age

3. Which of the following does the author NOT mention as a consequence of a large rise in global sea level?

A. The destruction of wetlands B. The flooding of cities

C. A more diverse marine population D. Severe storms

4. According to the passage, what is the relationship between carbon dioxide and the Earth’s climate?

A. Carbon dioxide, which is trapped in glacial ice, is released when warm temperatures cause the ice melt.

B. The greenhouse effect, which leads to the warming of the climate, is result of too much carbon stored in the

Earth’s crust.

C. Rain causes carbon dioxide to be washed out of the atmosphere and into the ocean.

D. An increase in atmospheric carbon dioxide results in the warming of the climate.

5. The word beneficial in the first paragraph is closest in meaning to

A. calm B. inviting C. thoughtful D. favorable

6. The word massive can be replaced to

A. wide B. huge C. dense D. thick

7. It can be inferred from the passage that the development of agriculture

A. preceded the development of animal husbandry.

B. withstood vast changes in the Earth’s climate.

C. did not take place during an ice age.

D. was unaffected by the greenhouse effect.

8. The word “this” in the third paragraph refers to

A. man’s upsetting the equation B. the reduction of the level of carbon dioxide.

C. a volcanic eruption D. the melting of the polar ice caps

9. The word “inhospitable” is closest in meaning to

A. imperfect. B. uninhabitable. C. unlikable D. cruel.

10. What is the main topic of the passage?

A. The possibility that the popular ice caps will melt

B. The coming of another ice age

C. Man’s effect on the carbon dioxide level in the atmosphere

33

D. The climate of the Earth over the years

Question 2. There are five extracts which have been removed from the following passage. Put an extract from A-G

in the correct gap 1-5 to complete the passage. There is two extracts you're not going to need. Write your answers

in the space provided. (5p.)

Children between the ages of 4 and 12 are the most common sufferers of sleepwalking, however, (1)……….,

somnambulism, as it is observed in adults, increases in severity and frequency with age. It is reported that more boys

sleepwalk than girls and it is between the ages of 11 and 12 that the most number of cases is reported. Also, many

children tend to grow out of it and because of this, it is surmised (2)………….that may relate to the physiology of the

younger body and mind that does not relate to adulthood.

Sleepwalking also seems to occur with more intensity or frequency in a person (3)………. not remember the incident

at all. It is also a common misconception that it is dangerous to wake a sleepwalker. It is, in fact, more dangerous to

not wake a sleepwalker. This is due to the actions that can be done by a sleepwalker. A person might drive a car, walk

out into traffic, try cooking, etc. All of these actions are highly dangerous to a person (4)…………what they are

doing.

This disorder can be just a mild annoyance to some but can also be a life-altering disorder to others. Sleepwalking is

more serious than aggressive so the danger lies more with self-injury than injuries to others. Sleepwalkers are not

allowed in the Armed Services at least partly due to the damage they can cause themselves and also partly to the fact

(5)……………… Treatments can range from relaxation techniques to anti-depressant medications.

A. that there are other sources for sleepwalking

B. who is sleep-deprived and a sleepwalker often will

C. that advise women to exercise for a good night’s sleeps

D. while pre-adolescents make up the larger number

E. who want to have an early night sleep

F. that they are around dangerous equipment

G. who is asleep and unaware of

Question 3. Match each of the following headings with its suitable paragraph. The first one (0: H) as an example

has been done for you. There are two extra headings which you do not need to use. Write your answers in the

space provided. (5p.)

A. Research holds the key to success B. New and improved techniques

C. A new kind of athlete D. New equipment has made a difference

E. Athletes are what they eat F. Personalized programs will help

G. The influence of drugs H. Is there a limit to record-breaking?

[0: ___H __]

A world record is every athlete’s dream, but the hard-won records of a few years ago are mostly just today’s

qualifying times. Roger Bannister’s famous four-minute mile of 1956 has been beaten by nearly 15 seconds, while

almost an hour and twenty minutes has been taken off the women’s marathon since 1953. ‘Faster, higher, stronger’ is

34

the Olympic motto, and today’s competitors continue to push back the boundaries of what the body can achieve. But

one wonders if this can continue.

[1: ______ ]

The last forty years have seen many important technological advances. For example, since the introduction of strong,

flexible fiberglass poles, over a meter has been added to the pole vault record. There have also been important

developments in the design the running shoe. And while a shoe won’t actually make someone run faster, modern

shoes do mean many more miles of comfortable, injury-free training.

[2: ______ ]

Pushing back the limits now depends more on science, technology and medicine than anything else. Athletic

technique, training programs and diets are all being studied to find ways of taking a few more seconds off or adding a

few more cm to that elusive world record. It seems that natural ability and hard work are no longer enough.

[3: ______ ]

The search to find more efficient ways of moving goes on. Analysis of an athlete’s style is particularly useful for

events like jumping and throwing. Studies show that long jumpers need to concentrate not on the speed of approach,

as once thought, but on the angle their bodies make with the ground as they take off. However, the rules governing

each sport limit advances achieved by new styles. For instance only one-footed takeoffs are allowed in the high jump.

[4: ______ ]

In the future, it should be possible to develop a more individual approach to training programs. Athletes will keep

detailed diaries and collect data to help predict the point when training becomes overtraining, the cause of many

injuries. If athletes fee all the information into a database, it may then be possible to predict patterns and to advise

them individually when they should cut down.

[5: ______ ]

Combining the right diet with a training program is vital. Athletes are continually searching for that special ‘go-faster’

ingredient, but apparently it’s still a battle to get them to drink sufficient liquid and to follow a balanced healthy diet

throughout all phases of training, competition and recovery. Diet in the period after an event is particularly important

and often neglected. An athlete who doesn’t replace all the liquid lost immediately after a hard run won’t be able to

repeat the performance at the same level 24 hours later.

Question 4. Fill each of the numbered blanks in the following passage with one suitable word. Write your answers

in the space provided. (10p.)

For over two hundred years, scholars have shown an interest in the way children learn to speak and understand their

language. Several small-scale studies were carried out, especially towards the end of the nineteenth century,

(1)_______ data recorded in parental diaries. But detailed, systematic investigation did not begin until the middle

decades of the twentieth century, when the tape recorder (2) _______ into routine use. This made it possible to keep a

permanent record of samples of child speech, so that analysts could listen repeatedly to obscure (3) _______, and thus

produce a detailed and accurate description. The problems that have (4) _______ when investigating child speech are

quite different from (5) _______ encountered when working with adults. It is not possible to carry out certain kinds of

experiments, because aspects of children’s cognitive development, such as their ability to (6) _______ attention or to

35

remember instructions, may not be sufficiently advanced. (7) _______ is it easy to get children to (8) _______

systematic judgments about language – a task that is virtually impossible below the age of three. Moreover, anyone

who has tried to make a tape recording of a representative sample of a child’s speech knows how frustrating this can

be. Some children, it seems, are innately programmed to (9) _______ off as soon as they notice a tape recorder (10)

_______ switched on.

PART IV: WRITING (20p.)

Question 1: Finish each of the following sentences in such a way that it is as similar as possible in meaning to the

sentence printed before it. (5p.)

Example: We expect he will arrive by nine o’clock

He is____________________________

Answer: He is expected to arrive by nine o’clock

1. The journalists only heard about the changes to the wedding plans when they arrived at the venue.

It was only ______________________________________________________

2. We only came to this restaurant because you insisted that we did so.

It was at _________________________________________________________

3. Arguing with her won’t get you anywhere.

It won’t do _______________________________________________________

4. The thief must have come in through the window.

The thief almost ___________________________________________________

5. What put me off the idea was simply how expensive it was going to be.

The sheer _________________________________________________________

Question 2. For each of the sentence below, write a new sentence as similar as possible in meaning to the original

sentence but using the word given. This word must not be altered in any way. (5p.)

Example: We couldn't find the cat anywhere. (NOWHERE )

Answer: The cat was nowhere to be found.

1. She is determined to become a doctor. (HEART)

_________________________________________________________________

2. She just pretended to welcome him, then quickly left the waiting room. (MOTIONS)

_________________________________________________________________

3. The success of the venture cannot be guaranteed. (SUCCEED)

_________________________________________________________________

4. We simply must pay them the whole amount before the end of the month. (ALTERNATIVE)

_________________________________________________________________

5. Ours is the only company allowed to import these chemicals. (MONOPOLY)

_________________________________________________________________

Question 3. Essay writing (10p.)

Do you agree or disagree with the following statement?

36

Face-to-face communication is better than other types of communication such as letters, e-mail,

or telephone calls.

Use specific reasons and details to support your answer. You should write about 250 words.

SỞ GIÁO DỤC – ĐÀO TẠO

HẢI DƯƠNG

KỲ THI CHỌN HỌC SINH GIỎI TỈNH

LỚP 12 THPT NĂM HỌC 2012- 2013

Thời gian làm bài: 180 phút

B. PHONETICS: (5 points)

Choose the word that has a different stress pattern from the others in the group:

16. A. responsibility B. originality C. accommodation D. mischievousness

17. A. appliance B. conscientious C. independent D. confidential

18. A. psychology B. environmental C. impossible D. photography

19. A. stimulate B. maximize C. interrupt D. register

20. A. appointment B. punishment C. publicity D. efficient

C. GRAMMAR – VOCABULARY – LANGUAGE FUNCTIONS:

I. Choose the word, phrase or expression which best completes each sentence: (15 points)

21. Tom: “I thought your performance last Sunday was wonderful.”

Laura: “………………………..”

A. Don’t tell a lie. I thought it was terrible.

B. You must be kidding. It was not as good as I had expected.

C. I completely agree with you. It was terrific.

D. No doubt!

22. Boy: “What is your greatest phobia?”

Girl: “………………………..”

A. I'm afraid not. B. Worms, definitely!

C. Probably people who smoke. D. I haven't made up my mind.

23. Jenny: “Thank you very much for your donation, Mr. Robinson.”

Mr. Robinson: “………………………..”

A. You can say that again. B. I see.

C. You are right. D. Delighted I was able to help.

24. I don’t think you have been watering the plants near the gate. The soil is ………………………..

A. as dry as rice B. as dry as a tile C. as dry as a bone D. as dry as wood

25. Susan was sad because she wasn’t invited to any social events. She felt ………………………..

A. left out B. turned out C. omitted out D. gone out

26. Most psychologists believe that the basic structure of an individual’s personality is………………………..

37

ĐỀ CHÍNH THỨC

A. well established extremely by the age of five

B. by the age of five it is extremely well established

C. by the age of five and well established extremely

D. extremely well established by the age of five

27. In most ……………………….. developed countries, up to 50% of ………………………..population

enters higher education at some time in their lives.

A. Ø / Ø B. the / Ø C. Ø / the D. the / a

28. I wish you ……………………….. me a new one instead of having it………………………..as you did.

A. would give / to repair B. gave / to repair

C. had given / to be repaired D. had given / repaired

29. Henry was really a silly boy when we were at high school. I still remember………………………..very stupid

questions.

A. him asking B. him to ask C. asking him D. his being asked

30. ………………………..the invention of the steam engine, most forms of transport were horse-drawn.

A. With reference B. Akin C. Prior to D. In addition to

31. No sooner ………………………..to marry Jack ………………………..to have serious doubts.

A. had Carol agreed / than she began

B. Carol has agreed / than she began

C. had Carol agreed / than she begins

D. had Carol agreed / than she had begun

32. Having been served dinner, ……………………...

A. the problem was discussed by the members of the committee.

B. the committee members discussed the problem.

C. it was discussed by the committee members the problem.

D. a discussion of the problem was made by the members of the committee.

33. The marathon runner ……………………...for nearly one hour and a half when she ……………………...to the

pavement.

A. has been running / collapses B. were running / collapsed

C. had been running / collapsed D. ran / had been collapsing

34. Son: “What is the process of ……………………..., Dad?”

Father: “Well, it involves the heating of liquid such as milk in order to kill harmful bacteria.”

A. industrialization B. pasteurization C. commercialization D. globalization

35. An artist ……………………...will do his best to express innocence and inexperience in the child’s face.

A. portraying a child B. who portray a child C. he portrays a child D. portrayed a child

II. Give the correct form of the words in brackets: (10 points)

36. Deforestation and excessive farming have ……………………... the soil.

37. The damage caused by the terrible storm two days ago was……………………... by

the government. The real figures go up every minute.

(POOR)

(ESTIMATE)

38

38. Barack Obama is the first President of the United States with ……………….…...

background.

39. From the hotel there is a……………………...view across the canyon.

40. The policeman examined the parcel ……………………... as he had no idea what it

could be.

41. It was very ……………………...of you to notice that.

42. In his …………………….., Mike smashed all the breakable items in the kitchen.

43. Before enrolling on a course, you should first ensure that it has been

……………………... by an officially recognized body.

44. Mr. John, who teaches us Latin, usually stresses the need for regular

……………………...

45. The government has yet to make an official ……………………...on the issue.

(RACE)

(BREATH)

(SUSPECT)

(OBSERVE)

(FURIOUS)

(VALID)

(ATTEND)

(PRONOUNCE)

III. Choose the underlined word or phrase in each sentence that needs correcting (5 points):

46. Dictionaries frequently explain the origin of the defined word, state its part of speech and indication its correct

use.

A B C D

47. The rings of Saturn are so distant to be seen from the Earth without a telescope.

A B C D

48. Children subjected to violence, exploitation , abuse and neglect are in risk of death, poor physical and

mental

A B C

health, HIV/AIDS infection, and educational problems.

D

49. It is important that cancer is diagnosed and treated as early as possible in order to assure a successful

cure.

A B C

D

50. A ten-thousand-dollars reward was offered for the capture of the escaped prisoner.

A B C D

D. READING:

I. Read the following passage and decide which option A, B, C or D best fits each space: (10 points)

Everyone has got two personalities - the one that is shown to the world and the other that is (51)

…………... and real. You don’t show your secret personality when you are (52)…………..., because you

can control yourself. But when you are asleep, your feeling position (53)…………...the real you. In a normal

(54)…………..., of course, people often change their position. The important position is the one you go to

sleep in.

If you go to sleep on your back, you are a very (55)…………...person. You usually believe people and

you accept new things or new ideas easily. You don’t like to make people sad, so you never express your

39

(56) …………... feeling. You are quite shy.

If you sleep on your stomach, you are a rather secretive person. You (57)…………...a lot and you

always easily become sad. You usually live for today not tomorrow. This means that you (58)

…………...having a good time.

If you sleep curled up, you are probably a very (59)…………...person. You have a low opinion of

yourself. You are shy and don’t like meeting people. You (60)………….....to be on your own. You are easily

hurt.

51. A. important B. serious C secret D. particular

52. A. awake B. active C happy D. honest

53. A. makes B. understand C changes D. shows

54. A. room B. bed C night D. body

55. A. independent B. open C talkative D. generous

56. A. real B. lonely C cheerful D. gentle

57. A. talk B. sleep C relax D. worry

58. A. regret B. enjoy C mind D. deny

59. A. strong B. healthy C nervous D. careful

60. A. pretend B. oppose C refuse D. prefer

II. There are 5 blanks in the passage below. From the words given in the box, choose the most

suitable for each blank. There are THREE extra words that you do not need to use: (5 points)

A. legal

B. community

C. responsibility

D. give back

E. serve

F. long-term

G. organizations

H. rare

Volunteering has many intangible benefits. It can help you (61) ……………………...to society, break

down barriers of misunderstanding or fear, explore personal issues and even have fun. It also has a

meaningful, positive impact on your (62)…………………….... But did you know that it can have many

benefits for you, too? You may have heard that volunteering helps you get into college, but keep in mind

they are not just looking for a list of (63)……………………... and dates. Colleges want to see a complete

picture of you and real examples of your commitment, dedication and interests.

Volunteering brings together a variety of people. Both the recipients of your volunteer efforts and

your co-workers can be rich sources of insight. For example, maybe you’ll learn about the (64)

……………………...profession from a former lawyer you visit at a convalescent center. Colleges pay

attention to your life inside and outside the classroom. Your extracurricular activities reveal a great deal

about you, such as what your interests are, whether you can manage your priorities and maintain a (65)

……………………... promise and how you’ve made an important contribution to something.

III. Read the passage and choose the correct answer: (10 points)

40

For many people who live in cities, parks are an important part of the landscape. They provide a

place for people to relax and play sports, as well as a refuge from the often harsh environment of a city.

What people often overlook is that parks also provide considerable environmental benefits.

One benefit of parks is that plants absorb carbon dioxide—a key pollutant—and emit oxygen, which

humans need to breathe. According to one study, an acre of trees can absorb the same amount of carbon

dioxide that a typical car emits in 11,000 miles of driving. Parks also make cities cooler. Scientists have

long noted what is called the Urban Heat Island Effect: building materials such as metal, concrete, and

asphalt absorb much more of the sun’s heat and release it much more quickly than organic surfaces like

trees and grass. Because city landscapes contain so much of these building materials, cities are usually

warmer than surrounding rural areas. Parks and other green spaces help to mitigate the Urban Heat

Island Effect.

Unfortunately, many cities cannot easily create more parks because most land is already being used

for buildings, roads, parking lots, and other essential parts of the urban environment. However, cities could

benefit from many of the positive effects of parks by encouraging citizens to create another type of green

space: rooftop gardens. While most people would not think of starting a garden on their roof, human beings

have been planting gardens on rooftops for thousands of years. Some rooftop gardens are very complex

and require complicated engineering, but others are simple container gardens that anyone can create with

the investment of a few hundred dollars and a few hours of work.

Rooftop gardens provide many of the same benefits as other urban park and garden spaces, but

without taking up the much-needed land. Like parks, rooftop gardens help to replace carbon dioxide in the

air with nourishing oxygen. They also help to lessen the Urban Heat Island Effect, which can save people

money. In the summer, rooftop gardens prevent buildings from absorbing heat from the sun, which can

significantly reduce cooling bills. In the winter, gardens help hold in the heat that materials like brick and

concrete radiate so quickly, leading to savings on heating bills. Rooftop vegetable and herb gardens can

also provide fresh food for city dwellers, saving them money and making their diets healthier. Rooftop

gardens are not only something everyone can enjoy, they are also a smart environmental investment.

66. Based on its use in paragraph 2, it can be inferred that mitigate belongs to which of the following word

groups?

A. exacerbate, aggravate, intensify B. obliterate, destroy, annihilate

C. allay, alleviate, reduce D. absorb, intake, consume

67. Using the information in paragraph 2 as a guide, it can be inferred that ..................................

A. cities with rooftop gardens are cooler than those without rooftop gardens

B. some plants are not suitable for growth in rooftop gardens

C. most people prefer parks to rooftop gardens

D. most people prefer life in the country over life in the city

68. According to the passage, the Urban Heat Island Effect is caused by the fact(s) that

..................................

a. cities are warmer than nearby rural areas

41

b. building materials absorb more of the sun’s heat than organic surfaces

c. building materials release the sun’s heat more quickly than organic surfaces

A. a. only B. a. and b. only C. b. and c. only D. a., b., and c.

69. Based on the information in paragraph 3, which of the following best describes the main difference

between parks and rooftop gardens?

A. Parks are expensive to create while rooftop gardens are not.

B. Parks are public while rooftop gardens are private.

C. Parks absorb heat while rooftop gardens do not.

D. Parks require much space while rooftop gardens do not.

70. The author claims all of the following to be the benefits of rooftop gardens except ..................................

A. increased space for private relaxation

B. savings on heating and cooling costs

C. better food for city dwellers

D. improved air quality

71. According to the author, one advantage that rooftop gardens have over parks is that they

..................................

A. decrease the Urban Heat Island Effect B. replenish the air with nourishing oxygen

C. do not require the use of valuable urban land D. are less expensive than traditional park

spaces

72. The author’s tone in the passage is best described as..................................

A. descriptive B. passionate C. informative D. argumentative

73. Which of the following pieces of information would, if true, most weaken the author's claim that rooftop

gardens are good for the environment?

A. Parks have many benefits that rooftop gardens do not share.

B. More pollution is produced during rooftop garden construction than rooftop plants can remove from

the air.

C. Extremely high winds atop tall city buildings can severely damage some plants.

D. The overall environmental benefits that result from driving less exceed those of planting a rooftop

garden.

74. Which of the following best describes the organization of the passage?

A. A hypothesis is stated and then analyzed.

B. A proposal is evaluated and alternatives are explored.

C. A viewpoint is established and then defended.

D. A thesis is presented and then supported.

75. It can be inferred from the passage that the author would most likely endorse a program that

..................................

A. permitted the construction of buildings in city park land provided they have rooftop gardens

B. extended discounts on plants to customers who use them to create rooftop gardens

42

C. offered free admission to schools willing to take their students on field trips to the city park

D. promised vacation getaways to cooler destinations for those trapped in the city at the peak of

summer

E. WRITING:

I. Finish each of the following sentences in such a way that it means exactly the same as the

sentence printed before it: (5 points)

76. Its lack of irregular verbs makes Esperanto a unique language.

Unlike………………………………….........................................………………………………………………….

77. Mr. Smith knew little about the Internet, so he didn’t invest into any computer companies.

Had…................................................................................................................................................................

78. The Board of Directors discussed the business in length, but came to no decision.

The Board of Directors

had ......................................................................................................................................

79. When she got to the party, everyone was dancing and singing.

On..................................................................................................................................................................

80. Barbara runs a successful company and she also manages to look after her five children.

Not only.........................................................................................................................................................

81. It is my opinion that you should spend more time practicing English.

If.....................................................................................................................................................................

82. Everyone was surprised that the famous actor had very little money when he died.

The famous actor..........................................................................................................................................

83. Success in the academic field depends on your ability to amass qualifications.

The more.....................................................................................................................................................

84. “Get out of my house or I will shoot you”, the woman shouted at the strange man.

The woman threatened.................................................................................................................................

85. What mainly distinguishes man from other animals is the power of speech.

Man………….................................................................................................................................................

II. After having attended a course on computing at ABC Information Technology Center for two

weeks, you notice that many things are worse than what were advertised by the Center. Write a

letter of complaint of about 150 words to the Director of the Center: (10 points)

Your letter should include:

- the reason for writing

- the problems you have encountered

- your suggestions and the actions you wish to be taken to resolve the problems

Trong bài viết thí sinh phải dùng tên và địa chỉ sau:

Pham Hai Nam

43

19 Ly Nam De Street, Hai Duong City

III. Violence among school students seems to be increasing. Write a passage of about 150 – 170

words to suggest what should be done to help solve this problem: (10 points)

SỞ GD&ĐT KỲ THI CHỌN HỌC SINH GIỎI CẤP TỈNH LỚP 12 THPT

QUẢNG BÌNH NĂM HỌC 2012 - 20013

Khoá thi: Ngày 11 tháng 10 năm 2012 ( 180 phút )

SECTION TWO: PHONETICS

Part 1. Pick up the word whose underlined part is pronounced differently from the others.

11. A. hobby B. comfort C. gone D. popular

12. A. massage B. message C. voyage D. dosage

13. A. naked B. wicked C. locked D. sacred

Part 2. Choose the word whose main stress is placed differently from the others in each group.

14. A. politics B. impolite C. volunteer D. afternoon

15. A. difficult B. portable C. element D. sincerely

SECTION THREE: LEXICO - GRAMMAR

Part 1. Choose the word or phrase (A, B, C or D) which best completes each sentence.

16. John: “Well it was nice talking to you, but I have to dash.” - Jane: “……………………….”

A. Well, another time. B. Yes, I enjoyed talking to you, too.

C. OK, see you. D. That's nice.

17. ………… pollution control measures are expensive, many industries hesitate to adopt them.

A. Although B. However C. Because D. On account of

18. ………….. Tom nor his brothers help in the housework.

A. Not B. Both C. Either D. Neither

19. I must congratulate you. You’ve …………….. a very good job.

A. done B. made C. finished D. worked

20. For ………….. reason is this meeting being held?

A. what B. why C. how D. that

21. I’ve been working ………….…. quite a lot of pressure lately.

A. in B. with C. on D. under

22. She loves London, ……………… in the spring.

A. mostly B. especially C. most D. specially

23. Make sure you end each sentence with a …………… .

A. dot B. point C. comma D. full stop

24. They travelled to the capital city of Scotland by the most ……..…….. route.

A. easy B. direct C. straight D. unique

25. He failed in the election just because he ……...……. his opponent.

A. overestimated B. underestimated C. understated D. undercharged

44

26. Which …………… agency do you work for?

A. ads B. advertised C. advertising D. advertisement

27. His face looks …………….., but I can’t remember his name.

A. similar B. alike C. memorable D. familiar

28. There is a …………….. of skilled craftsmen in the industry.

A. want B. fault C. lack D. need

Part 2. Put the verbs given in brackets into the appropriate tenses or forms.

29. This is an awful hotel. I wish we (go) …………………. to The Grand instead.

30. My suggestion is that your brother (see) ……………….. the dentist.

31. Please wait a minute! My boss is busy (write) ………………… something.

32. (spend) ……………… all their money, they decided to go home.

33. Peter painted the room black. It looks dark and dreary. He (choose) ….… a different colour.

34. Nowadays children would prefer history (teach) ………………. in more practical ways.

35. Tell him to come at 5 p.m. because I (finish) ……………… my work by then.

Part 3. Read the text below. Use the words given in capitals at the end of each line to form a word that fits in the

space in the same line.

Part 4. Choose the underlined word or phrase in each sentence that needs correcting.

46. Tom said that if he had to do another homework tonight, he would not be able to attend the

A B C D

concert.

47. Stand up straight, breathe deeply, hold your head up, and no look down!

A B C D

48. It spent a long time to travel to the skiing resort but in the end, we got there.

A B C D

49. Paris has been well-known about its famous monuments, beautiful music, and wonderful

45

EXERCISEExercise is one of the best ways of keeping depression away. It

improves your body and your mind and (36) …………. you to perform better in the work place and at home.

Proper (37) ……..…… is essential if you want to get the most from exercise and you should also take into (38) ……..……… your heart rate. It can be (39) ……….…….. to do too much, which is why all good fitness instructors emphasise the (40) ………….… of “listening to your body.”

When you first start, you should use good (41) ……….. because it is easy to make the mistake of using the equipment (42) …………. or doing too much at one time. Start slowly and build up gradually.

Exercise should not be seen as a (43) .…………… task; it can be as easy as a quick walk. To increase your fitness (44) ……..…….., exercise for 20 minutes a day, 4 to 6 times a week and you will notice a (45) …………… in your body and mind in a few weeks.

ABLE

BREATHCONSIDERHARMIMPORTANT

JUDGECORRECT

DEMANDSTEADY

DIFFERENT

A B C

restaurants for over 100 years.

D

50. Adult humans have more than a trillion cells in his bodies.

A B C D

SECTION FOUR: READING COMPREHENSION

Part 1. Read the text below and think of the word which best fits each gap. Use only ONE word in each gap.

SPECTATOR SPORTS

A surprising number of popular spectator sports, for example football or basketball, started in Europe or the

USA in (51) ………. nineteenth century. This did not happen by chance. It was the result of changes in the way

people lived in those places at that time.

Until then (52) …..……. people lived in the country than in towns. They worked in small groups and had (53)

……..… regular time off. All this changed with the growth of factories and industry in the nineteenth century, first in

Europe and then in the USA. (54) ……….. the first time most people began to live in towns, and they found

themselves with regular free time. They had more leisure time than ever before.

This resulted in the need for organized entertainment. Suitable games developed or (55) ………. invented,

typically team games, in (56) ……….. the crowded could take sides and become involved. This gave people some of

the entertainment they needed in (57) ………….. free time.

The recent explosion in TV, with the introduction of satellite and cable channels, (58) …….…. caused an

increase in demand for sports as entertainment. The money TV has brought to games (59) ……….…. as football,

tennis and baseball means that spectator sports (60) ……….. certainly go on playing an important part in our lives.

Part 2. Choose the word that best fits each of the blanks in the following passage. Choose A, B, C or D to indicate

your answer.

How men first learnt to (61) ………... words is unknown; in other words, the origin of language is a (62) ….

….... . All we really know is that men, unlike animals, (63) .…….….. invented certain sounds to express thoughts and

feelings, actions and things so that they could communicate with each other; and that later they agreed (64)

………….. certain signs, called letters, which could be combined to represent those sounds, and which could be

written down. These sounds, (65) ………….. spoken or written in letters, are called words.

Great writers are those who not only have great thoughts but also express these thoughts in words that (66)

………….. powerfully to our minds and emotions. This charming and telling use of words is what we call literary

(67) …………... . Above all, the real poet is a master of words. He can (68) ………….. his meaning in words which

sing like music, and which by their position and association can (69) ………….. men to tears. We should, therefore,

learn to choose our words carefully, (70) …………… they will make our speech silly and vulgar.

61. A. invent B. create C. make D. discover

62. A. story B. secret C. mystery D. legend

63. A. whatever B. however C. somewhat D. somehow

64. A. at B. upon C. for D. in

65. A. if B. however C. whether D. though

46

66. A. interest B. appeal C. attract D. lure

67. A. prose B. work C. form D. style

68. A. carry B. convey C. transfer D. transmit

69. A. take B. send C. break D. move

70. A. or B. so C. although D. because

Part 3. Read the following passage and choose the letter A, B, C or D to indicate the correct answer to each

question.

Reading to oneself is a modern activity which was almost unknown to the scholars of the classical and

medieval worlds, while during the fifteenth century the term “reading” undoubtedly meant reading aloud. Only during

the nineteenth century did silent reading become commonplace.

One should be wary, however, of assuming that silent reading came about simply because reading aloud was

a distraction to others. Examinations of factors related to the historical development of silent reading have revealed

that it became the usual mode of reading for most adults mainly because the tasks themselves changed in character.

The last century saw a steady gradual increase in literacy and thus in the number of readers. As the number of

readers increased, the number of potential listeners declined and thus there was some reduction in the need to read

aloud. As reading for the benefit of listeners grew less common, so came the flourishing of reading as a private

activity in such public places as libraries, railway carriages and offices, where reading aloud would cause distraction

to other readers.

Towards the end of the century, there was still considerable argument over whether books should be used for

information or treated respectfully and over whether the reading of materials such as newspapers was in some way

mentally weakening. Indeed, this argument remains with us still in education. However, whatever its virtues, the old

shared literacy culture had gone and was replaced by the printed mass media on the one hand and by books and

periodicals for a specialised readership on the other.

By the end of the twentieth century, students were being recommended to adopt attitudes to books and to use

reading skills which were inappropriate, if not impossible, for the oral reader. The social, cultural and

technological changes in the century had greatly altered what the term “reading” implied.

71. Reading aloud was more common in the medieval world because ……………….. .

A. people relied on reading for entertainment

B. silent reading had not been discovered

C. there were few places available for private reading

D. few people could read to themselves

72. The development of silent reading during the last century indicated ………………... .

A. an increase in the average age of readers B. an increase in the number of books

C. a change in the nature of reading D. a change in the status of literate people

73. Silent reading, especially in public places, flourished mainly because of …………… .

A. the decreasing need to read aloud B. the development of libraries

C. the increase in literacy D. the decreasing number of listeners

74. The phrase “oral reader” in the last paragraph mostly means “a person who ……………. .”

47

A. is good at public speaking B. practises reading to an audience

C. takes part in an audition D. is interested in spoken language

75. Which of the following statements is NOT TRUE according to the passage?

A. Reading aloud was more common in the past than it is today.

B. The decline of reading aloud was wholly due to its distracting effect.

C. Not all printed mass media was appropriate for reading aloud.

D. The change in reading habits was partly due to the social, cultural and technological

changes.

SECTION FIVE: WRITING

Part 1. Finish each of the following sentences in such a way that it means exactly the same as the

sentence printed before it.

76. It was the goalkeeper who saved the match for us.

→ If it hadn’t…………………………………………………..

77. It is thought that the accident was caused by human error.

→ The accident………………………………………….……..

78. I think the last time I saw you was at Diana’s house-warming party.

→ I don’t think………………………………………………….

79. I gave Tom the message, but he already knew about it.

→ I needn’t……………………………………………………...

80. I’ve forgotten that singer’s name but she’s very well-known.

→ That singer,…………………………………………….…….

81. It is essential that no one is told about our plans.

→ You…………………………………………………………..

82. It is your duty to tell him what to do.

→ You are ……………………………………………………..

83. Do you have a good relationship with your boss?

→ Are you…………………………………………………..…?

84. Under no circumstances should you phone the police.

→ The last ………………………………………………….…..

85. The permit expires at the end of this month.

→ The permit is not…………………………………………….

Part 2. Complete the second sentence so that it has a similar meaning to the first

sentence using the word given. DO NOT CHANGE THE WORD GIVEN.

86. Susan said that she didn’t want to go out that evening. (RATHER)

→ Susan said that ……………………………………. out that evening.

87. When was the last time you saw Robert? (LONG)

→ How ……………………………..……... you last saw Robert?

88. Would you like to see the photos I took? (HAVE)

48

→ Would you like …………………….………..….…. the photos I took?

89. They share a lot of hobbies and interests. (COMMON)

→ They have a lot of ……………………………..………… .

90. I daren’t turn on the television because the baby might wake up. (FEAR)

→ I daren’t turn on the television …………………..…… the baby up.

Part 3. Essay writing

Do you agree or disagree with the following statement? Parents are the best teachers. Use specific reasons

and examples to support your answer. You should write at least 200 words.

……………………………………………………………………………………………………………………

……………………………………………………………………………………………………………………………

------THE END------

SỞ GD&ĐT KỲ THI CHỌN HSG CẤP TỈNH LỚP 12 THPT

QUẢNG BÌNH NĂM HỌC 2012-2013

Môn thi: TIẾNG ANH (Vòng 2)

ĐỀ THI CHÍNH THỨC Khóa thi: Ngày 11 tháng 10 năm 2012

Thời gian làm bài: 180 phút (không kể thời gian giao đề)

SECTION TWO: LEXICO - GRAMMAR

Part 1. Choose the word or phrase (A, B, C or D) that best completes each sentence.

16. The company has just got a big order and the workers are working round the ……………….

A. day B. clock C. hour D. night

17. ……………. long, I'm sure you will be speaking English ………………. a native speaker.

A. After/ like B. Before/ like C. After/ with D. Before/ with

18. I know you’re annoyed, but you must try to control your ….………….. .

A. blood B. storm C. explosion D. temper

19. Everyone in our family says best wishes to …………….. in the early morning of the first day

of the New Year.

A. ourselves B. themselves C. one another D. the others

20. You will have to go for an interview tomorrow, but don't worry. It's just a …………… .

A. form B. format C. formation D. formality

21. Good restaurants serving traditional English food are very hard to ………………… .

A. come into B. come by C. come to D. come at

22. Do you need to give your speech another ..................... or do you already know it by heart?

A. break-down B. check-up C. run-through D. mix-up

23. Mr. Jones knew who had won the contest, but he kept it under his ………………. until it was

announced publicly.

A. cap B. tongue C. hat D. umbrella

49

SỐ BÁO DANH:

Part 2. Fill in each of the blanks with the correct form of the word in the bracket.

24. We can look forward to a period of ………...……. . (PROSPER)

25. It is said the ……………..…. of a Swiss watch is perfect. (PRECISE)

26. In nursing, women tend to ……………..…. men by four to one. (NUMBER)

27. We have to decide to interview only the best six ……..……..… for the job. (APPLY)

28. Unless we do research on ……….…….. energy, wind power and tidal power, our fossil fuels

will run out. (SUN)

Part 3. Fill in each blank with an appropriate preposition or particle to make sentences.

29. I’m faithful …………….….. my principle.

30. He looks like his father ………………………. appearance.

31. Your suspicions are quite ................................. foundation.

32. You can’t sit ............................. and do nothing like that.

33. I'd like to thank the speaker ………….……….. behalf of all the attenders.

Part 4. Complete each sentence with the correct form of the phrasal verbs in the box. Use each once only.

There are two extra phrasal verbs which you do not need to use.

take after go off come across do without come in for be with

sell out look into take up drop in on cut down on get in

34. We ……………….. tea and drank coffee instead.

35. Any time you're in the area, feel free to ……………….. us.

36. Tom: “Does Tan ……………….. his father or mother?”

Jane: “Well, he looks just like his father, but has his mother nature”.

37. You should ……………… smoking if you can't stop completely.

38. We'll give you our decision when we have had time to ……………. the matter.

39. Don't let your windows open when you are away from home or a burglar might ……………

40. I'm not very fit, so I’ve decided to ……….……. an active hobby such as squash or jogging.

41. There were clashes between friend and myself at times but I ……... really …........ him every

serious trouble.

42. The bomb …………….. with a loud bang which could be heard all over the town.

43. They are going to ……………… a lot of criticism for increasing bus fares by so much.

Part 5. Read the text below and look carefully at each line. Some of the lines are correct, and some have a word

which should not be there. If a line is correct, put a tick (√) by the number. If a line has a word which should not

be there, write the word by the number.

0. ..√…

00. will

44. …..

45. …..

50

Cars became popular as a quick and comfortable way of getting around. This is still true when you will drive along a quiet country road or a modern motorway. As far as getting from one place to another in the city is concerned, it is a different story. Whenever I want to get up anywhere in a hurry, I leave the car at home and go on foot. It often turns out to be much more quicker. I still make the mistake now and again of thinking the car is an efficient means of a transport. The other day my wife was

46. …..

47. …..

48. …..

49. …..

50. …..

51. …..

52. …..

53. …..

SECTION THREE: READING COMPREHENSION

Part 1. Read the following passage, and then choose the correct answers (A, B, C or D).

Because writing has become so important in our culture, we sometimes think of as more real than speech. A

little thought, however, will show why speech is primary and writing secondary to language. Human beings have been

writing (as far as we can tell from surviving evidence) for at least 5000 years; but they have been talking for much

longer, doubtless ever since there have been human beings.

When writing did develop, it was derived from and represented speech, albeit imperfectly. Even today, there

are spoken languages that have no written form. Furthermore, we all learn to talk well before we learn to write; any

human child who is not severely handicapped physically or mentally learns to talk: a normal human being cannot be

prevented from doing so. On the other hand, it takes a special effort to learn to write; in the past many intelligent and

useful members of society did not acquire the skill, and even today may who speak languages with writing systems

never learn to read or write, while some who learn the rudiments of those skills so only imperfectly.

To affirm the primacy of speech over writing is not, however, to disparage the latter. One advantage writing

has over speech is that it is more permanent and makes possible the records that any civilization must have. Thus, if

speaking makes us human, writing makes us civilized.

54. The author of the passage argues that ……………………..

A. writing has become too important in today’s society.

B. speech is more basic to language than writing.

C. everyone who learns to speak must learn to write.

D. all languages should have a written form.

55. According to the passage, writing ……….…

A. represents speech, but not perfectly.

B. is imperfect, but less so than speech.

C. developed from imperfect speech.

D. is represented perfectly by speech.

56. In the author’s judgment……………………....

A. writing has more advantages than speech.

B. speech is essential but writing has important benefits.

C. speech conveys ideas less accurately than writing does.

51

feeling a bit under the weather. She had been having terrible headaches for some long time and she decided she couldn’t take it any more and asked from me to give her a lift to the doctor, whose surgery is in the center part of a town. We live in a suburb in the old quarter of the city and it is twenty minutes away on foot. On the way back, however, it is all up hill and I must to admit it can be exhausting, especially on a hot day. Reluctantly I got the car out of the garage and we set it off, muttering about the wonders of taxis. My heart sank as we hit the first traffic jam - I knew we were beginning a long journey.

D. writing is more real than speech.

57. In order to show that learning to write requires effort, the author gives the example of …….

A. people who learn the rudiments of speech .

B. people who speak many languages.

C. intelligent people who couldn’t write.

D. severely handicapped children.

58. According to the author, one mark of civilized society is that it ………....…

A. affirms the primacy of speech over writing.

B. affirms the primacy of writing over speech.

C. teaches its children to speak perfectly.

D. keeps written records.

Part 2. Read the text below and think of the word which best fits each space. Use only ONE word in each space.

A recent university research project investigated the attitudes of postgraduate science students (59) ………..

…… the learning of English vocabulary. The results were surprising. I’ll mention three of them.

Firstly, most of the students think that (60) …………....… every word in English has just one meaning. This

is, of course, completely contrary to the facts. A glance at any English dictionary will show this. The student will

frequently find seven or eight meanings listed (61) …….…..… quite “simple” words.

Why, then, have these students made such a mistake? One reason may be that they are all science students.

Scientists try to use words in their subject 62) ………..……. have one meaning, and one meaning only. Another

reason, of course, could be the way in which these students were taught. They may have used vocabulary lists when

they first (63) ……………… English. On one side of the page is the word in English; on the other side, a single word

in the student’s native language.

The second attitude that (64) ……………... from the findings is equally mistaken. Practically all the students

think that every word in English has an exact translational equivalent. Again, this is far from the truth. Sometimes one

word in English can only be (65) ………….….. by a phrase in the student’s native language. There are other

difficulties in translation which we won’t mention here. Certainly, the idea of one word for one word translation

process is completely false. Translation machines, which tried to work on this principle, failed completely.

The third result of the (66) ………….…. showed another error in the students’ thinking. They believe that as

soon as they know the meaning of a word, they’re in a position to use it correctly. This is (67) …………..… for any

language but is perhaps particularly false for English. The student has to learn when to use a word as well as to know

what it means. Some words in English mean almost the same but they can only be used in certain (68) ……………... .

What then, is the best way to increase one’s vocabulary? This can be answered in three words: observation, imitation

and repetition.

Part 3. You are going to read a magazine article. Seven paragraphs have been removed from the article. Choose from

the paragraphs A-H the one which fits each gap (69-75). There is one extra paragraph which you do not need to use.

A. As a result, the impact on the instrumental curriculum and the measures used to assess progress through it

will be such that they will need to adapt to maintain their relevance for a broader sector of the population. Ultimately,

they will need to encompass a wider range of musical skills.

52

B. For all these reasons, there is likely to be a continuing demand for instrumental teaching in the short term.

What about the longer term picture? Is there likely to be a shift in focus and, if so, what direction will it take?

C. I would respond to the latter question on a positive note. The music industry is one of the major generators

of income in Britain and musical skill and talent will continue to be important in preparing individuals to work in a

variety of professions, in particular those related to the media.

D. This represents a fundamental change from traditional practice and it will be accompanied by the need to

respond to demands for public accountability. Viewed positively, this should provide an opportunity for all those

involved in music education to demonstrate the high quality of music tuition available.

E. There seems little doubt that the widening access to music is likely to continue, fulfilling as it does so

many human needs. On a national level, no major state occasion is without music. For individuals, it provides

opportunities for numerous activities, formal and informal.

F. One of the best-recognised functions is that of providing an outlet for emotional expression. Its influence

on our moods can be therapeutic. It provides a means of communicating which goes beyond words and provides us

with shared unspoken understandings.

G. Developing in parallel with this trend is a likely increase in the number of people, across the whole age

range, who wish to actively participate in music making. Such activities are likely to be community based and will

reflect the musical traditions of that community whatever they may be.

H. While these results are still to be successfully replicated, other data from Europe has indicated that an

increase in .group music lessons can have positive effects on social relationships in school and on concentration in

young children and those with behavioural difficulties.

Music - The Challenge Ahead

Technological advances continue to transform our lives at work, at home and in our leisure activities. Susan

Hallam discusses their impact on music in Britain.

In the latter part of the 20th century, we saw a rapid increase in the opportunities available for listening to

music through radio, TV, records, tapes, CDs, videos and a rapidly developing range of multi-media techniques.

Along with this, there has been a decline in the performance of live music and in the full-time employment

opportunities for professional musicians.

(69) ………………….

Indeed, a society without music is surely unthinkable and it seems that the issue is not whether there will be

music in the 21st century but what the nature of that music will be; and also whether there will be a continued

perceived need for people to learn to play musical instruments.

(70) …………………

In addition to its vocational significance, there is a growing body of evidence that playing an instrument may

be beneficial to the development of skills at an earlier stage. Research in the USA has suggested that listening to or

actively making music has a direct positive effect on spatial reasoning, one aspect of the measurement of intelligence.

(71) ………………..

Taking the idea behind such findings one step further, current research is investigating to what extent playing

an instrument may even encourage the development of transferable skills. For instance, the need to practise regularly

53

may assist in the acquisition of good study habits and focused concentration; playing in concerts may encourage

habits of punctuality and good organization.

(72) …………………

While there are many possible scenarios, I believe that two possibilities are likely. Firstly, the kinds of music

to which people will listen will become more diverse. New genres will develop which will integrate different styles.

Secondly, there will be an increase in the use of technology to compose and perform music. This will widen access to

composition as there will be less reliance on technical skill but at the same time, it is likely to further reduce the need

for live performance and musicians whose role is solely related to it.

(73) ……………….

If this vision of the future is to be realised, what does the music profession need to do in preparation? The

focus of instrumental tuition will need to change. Ways will need to be found to enable more people to learn to play a

range of instruments, throughout their life span.

(74) …………….…

Crucial to the success of the process will be the training of musicians. They will need to be able to motivate,

inspire and teach learners of all ages, develop skills for working with large and diverse groups and acquire the

communication, social, entrepreneurial and management skills necessary for community work.

(75) ………..……...

Finally, we need to strive towards raising the profile of music itself. Music plays a crucial role in our lives but

all too often it is taken for granted. Those involved in the music profession at all levels need to work actively together

to ensure that this changes.

SECTION FOUR: WRITING

Part 1. Complete the second sentence so that it has a similar meaning to the first sentence, using the word given.

DO NOT CHANGE THE WORD GIVEN. You must use between three and five words, including the word given.

76. I never have enough money. (SHORT)

I …………………………………

77. I’ve got to get a new computer. (NEED)

What I really .................................... new computer.

78. I can’t describe people as well as you can. (BETTER)

You’re .................................................. I am.

79. It was easy for us to get tickets for the concert. (NO)

We ............................................. tickets for the concert.

80. I hate to watch late night films on TV. (STAND)

I …………………………… late night films on TV.

81. The players did their best, so the manager thanked them. (FOR)

The manager …………………….………….. their best.

82. He surprised me very much when he said he loved me. (BREATH)

He …………………………….. when he said he loved me.

83. She couldn’t finish the race because she was injured. (PREVENTED)

54

Her injury …….…………………………………… the race.

84. “It’s not worth worrying about the past”, I told him. (POINT)

I told him that there was .................................. about the past.

85. While I was driving, I realized that the car wasn’t working properly. (WRONG)

While I was driving, I realized that .............................................. the car.

Part 2. Rewrite each sentence so that it has the same meaning, and contains the word given in capitals. Do not

change the word in any way.

86. Nobody helped me at all. (FINGER)

Nobody …………………………………………….

87. Money is of little value on a desert island. (COUNTS)

Money ………………………………………………………….

88. I tried to talk to Jack about the problem but he was too busy. (WORD)

I tried ……………………………………………………………..

89. I don't mind whether we have the meeting today or tomorrow. (MAKES)

It …………………………………………………………………..

90. As far as I know he is still working in HoChiMinh city. (KNOWLEDGE)

To ………………………………………………………………….

Part 3. Graph writing

The graph shows Internet Usage in Taiwan by Age Group, 1998-2000. Summarise the information by

selecting and reporting the main features, and make comparisons where relevant. You should write at least 150

words.

TỈNH ĐẮK LẮK NĂM HỌC 2011 - 2012

Thời gian làm bài: 150 phút (không kể thời gian giao đề)

I. PHONETICS (1/20 points)

55

ĐỀ CHÍNH THỨC

Part 1: Circle the word whose underlined part is pronounced differently from that of the others in each

group. Circle A, B, C or D to indicate your answer.

1. A. arrow B. loud C. moldy D. poultry

2. A. basilisk B. bison C. basic D. basin

3. A. subtlety B. indebtedness C. bombard D. combing

4. A. benevolent B. content C. molecules D. technique

5. A. conscience B. bronchitis C. shuttle D. chauffeur

Part 2: Choose the word whose syllable is stressed differently from that of the others in each line. Circle A, B, C or

D to indicate your answer.

6. A. comic B. clementine C. climax D. thermonuclear

7. A. diligent B. dimension C. action D. innate

8. A. characterize B. absence C. datum D. charcoal

9. A. solicitor B. separately C. spacious D. sequence

10. A. parachute B. armchair C. accent D. accidentally

II. LEXICO – GRAMMAR (6/20 points)

Part 1: Choose the word or phrase that best completes each sentence. Circle A, B, C or D to indicate your answer.

11. Assembly lines are useful for producing a large _____ of identical products.

A. quality B. quantity C. quandary D. qualification

12. Only the _____ of the building is going to be remodeled.

A. insides B. interior C. indoors D. inner

13. Whether the sports club survives is a matter of complete _____ to me.

A. indifference B. disinterest C. importance D. interest

14. After years of neglect there was a huge _____ program to return the city to its former glory.

A. restoration B. preservation C. conservation D. refurbishment

15. The assistant suggested _____ the next day when the manager would be there.

A. we are coming back B. to come back C. we will come back D. we came back

16. I never get a _____ of sleep after watching a horror film.

A. wink B. blink C. night D. ounce

17. As it was Christmas, the _____ at church was much larger than usual.

A. audience B. convention C. congregation D. grouping

18. The sheep were huddled into a _____ to protect them from overnight frosts.

A. cage B. kennel C. hutch D. pen

19. The jury _____ the defendant “not guilty”.

A. gave B. returned C. subscribed D. found

20. Many _____ crafts such as weaving are now being revived.

A. customary B. habitual C. traditional D. ordinary

21. He managed to finish his thesis under the _____ of his tutor.

56

A. guidance B. help C. aid D. assistance

22. Mr. Henry was given a medal in _____ of his service to his country.

A. gratitude B. knowledge C. recognition D. response

23. Everyone knows about pollution problems, but not many people have _____ any solutions.

A. thought over B. come up with C. looked into D. got round to

24. You _____ as well seek for a fish in the tree as try to do that.

A. must B. would C. should D. might

25. _____ calculations have shown that the earth’s resources may run out before the end of the next century.

A. Raw B. Rude C. Crude D. Blunt

26. By the time you receive this letter, I _____ for China.

A. will have left B. have left C. would have left D. will leave

27. Prizes are awarded _____ the number of points scored.

A. resulting from B. adding up C. presented to D. according to

28. The needs of gifted children in schools have long been _____ neglected.

A. dolefully B. woefully C. idly D. pathetically

29. I must take this watch to be repaired; it _____ over 20 minutes a day.

A. increases B. gains C. accelerates D. progresses

30. It had been a trying afternoon, _____ at about six o’clock in the television breaking down.

A. culminating B. leading C. arriving D. finalizing

Part 2: Write the correct FORM of each bracketed word in the numbered space provided in the column on the

right. (0) has been done as an example.

According to some (0) _____ (SCIENCE), high-risk sports can be

particularly (31) _____ (VALUE) for certain types of people. Such

activities help them to learn that being (32) _____ (FRIGHT) doesn’t mean

that they have to lose control. The recent fashion for jumping from bridges

attached to a (33) _____ (LONG) of elastic rope, known as “bungee

jumping”, has now been tried by over one million people (34) _____

(WORLD) and interest in it is continuing to grow.

Before the special elastic rope (35) _____ (TIGHT) around them,

jumpers reach speeds of nearly 160kph. First-timers are usually too (36)

_____ (TERROR) to open their mouths, and when they are finally (37)

_____ (LOW) safely to the ground, they walk around with broad smiles on

their faces, saying (38) _____ (REPEAT) how amazing it was. However,

for some people, it is only the (39) _____ (EMBARRASS) of refusing to

jump at the last minute that finally persuades them to conquer their fear of

(40) _____ (HIGH) and push themselves off into space.

scientists

31. _____________________

32. _____________________

33. _____________________

34. _____________________

35. _____________________

36. _____________________

37. _____________________

38. _____________________

39. _____________________

40. _____________________

57

Part 3: The passage below contains 10 mistakes. Underline the mistakes and write their correct forms in

the space provided in the column on the right. (0) has been done as an example.

Traditional, mental tests have been divided into two types.

Achievement tests are designed to measure acquiring skills and knowledge,

particularly those that have been explicitness taught. The proficiency exams

required by few states for high school graduation are achievement tests.

Aptitude tests are designed and measure a person’s ability to acquire new

skills but knowledge. For example, vocation aptitude tests can help you

decide whether you would do better like a mechanic or musician. However,

all mental tests are in some sense achievement tests because they assumption

some sort of past learning or experience with certainly objects, words, or

situations. The difference between achievement and aptitude tests is the

degree and intention use.

0. traditional → traditionally

41. _____________________

42. _____________________

43. _____________________

44. _____________________

45. _____________________

46. _____________________

47. _____________________

48. _____________________

49. _____________________

50. _____________________

Part 4: Fill in each of the gaps with the correct preposition or particle. Write your answers in the

numbered spaces provided below the passage.

The show was fully booked (51) _____ for weeks, and when it opened last night, the public poured (52)

_____ and very soon the London Arts Center was packed (53) _____. But why? What did they come to see? They

came to see human beings take (54) _____ circus animals, men in cat suits who stood (55) _____ for real lions and

tigers. The show was put (56) _____ by its creators to protest (57) _____ traditional circuses and to send a message

about cruelty to animals. The show was timed to tie (58) _____ with the National Protection of Animals Week. It was

a good idea, but the standard of the performances was third-rate and an embarrassing number of people simply walked

(59) _____ before it ended. There were some amusing moments when the performers sent (60) _____ typical circus

folks, but overall it was a dismal show. Despite the large turnout for the show’s first night, I doubt it will attract many

people during the rest of its seven-day run.

Part 5: Complete each sentence with the correct form of ONE of the phrasal verbs below. Write

your answers in the numbered spaces provided below the passage. Each verb is used only once.

try out slip up carry on get by put out

take after get down look up go through turn down

61. If you’re finding it difficult to _____ on your salary, why don’t you ask for a raise?

62. I know what you’re _____ and I feel really sorry for you.

63. In many ways you _____ your father.

64. If you _____ you’ll get into trouble.58

65. I proposed to her but she _____ me _____.

66. You’d better _____ your cigarette because smoking isn’t allowed in here.

67. If you _____ working so hard, you’ll make yourself ill.

68. Stop worrying about it. Don’t let this failure ____ you _____.

69. The car’s in quite good condition but you can _____ it _____ before you make any decision to buy,

70. When I was in New York, I was able to _____ several old friends I hadn’t seen for years.

Part 6: Insert A, AN, THE or Ø (zero article) where necessary. Write your answers in the numbered spaces

provided below the passage.

Suddenly (71) _____ blackbird flew to (72) _____ top of (73) _____ beach. She perched way up on (74) _____

topmost twig that stuck up thin against (75) _____. Then she commenced to sing.

Her little black body seemed only (76) _____ tiny dark speck at that distance. She looked like (77) _____ old

dead leaf. But she poured out her song in (78) _____ great flood of rejoicing through (79) _____ whole forest. And

(80) _____ things began to stir.

III. READING COMPREHENSION (4/20 points)

Part 1: Read the following passage and decide which answer (A, B, C or D) best fits each gap. Circle A, B,

C or D to indicate your answer.

The ability to weep is a uniquely human form of emotional response. Some scientists have suggested that

human tears are (81) _____ of an aquatic past – but this does not seem very likely. We cry from the moment we enter

this world, for a number of reasons. Helpless babies cry to persuade their parents that they are ill, hungry or

uncomfortable. As they (82) _____, they will also cry just to attract parental attention and will often stop when they

get it.

The idea that having a good cry do you (83) _____ is a very old one and now it has scientific validity since

recent research into tears has shown that they (84) _____ a natural painkiller called enkaphalin. By fighting sorrow

and pain this chemical helps you feel better. Weeping can increase the quantities of enkaphalin you (85) _____.

Unfortunately, in our society we impose restrictions upon this naturally (86) _____ activity. Because some

people still regard it as a (87) _____ of weakness in men, boys in particular are admonished when they cry. This kind

of repression can only increase stress, both emotionally and physically.

Tears of emotion also help the body (88) _____ itself of toxic chemical waste, for there is more protein in them

than in tears resulting from cold winds or other irritants. Crying comforts, calms and can be very enjoyable – (89)

_____ the popularity of highly emotional films which are commonly (90) _____ “weepies”. It seems that people enjoy

crying together almost as much as laughing together.

81. A. witness B. evidence C. result D. display

82. A. evolve B. change C. develop D. alter

83. A. better B. fine C. good D. well

84. A. contain B. retain C. hold D. keep

85. A. construct B. achieve C. provide D. produce

59

86. A. curing B. treating C. healing D. improving

87. A. hint B. symbol C. feature D. sign

88. A. release B. rid C. loosen D. expel

89. A. consider B. remark C. distinguish D. regard

90. A. named B. entitled C. subtitled D. called

Part 2: Read the following passage and fill the blank with ONE suitable word. Write your answer in the

space provided below the passage.

New technologies, like all technologies, are morally neutral. (91) _____ their advent makes the world a better

place or not depends on the uses to which they are (92) _____. And that, (93) _____ turn, depends upon the decisions

of many people, especially of politicians, managers, trade (94) _____ leaders, engineers and scientists. The new

technologies, cheap, flexible, dependent on knowledge and information as their main input, can (95) _____ human

being from many of their current constraints for example constraints of resources and geography. (96) _____ the new

technologies could also (97) _____ those with power to control their fellow citizens even more effectively than in the

(98) _____ efficient dictatorships of the past. The new technological society will (99) _____ colossal demands on our

imagination and ingenuity and on the capacity (100) _____ our institutions to respond to new challenges.

Part 3: Read the following passage and answer the questions from 101 to 110.

Legend has it that sometime toward the end of the Civil War (1861-1865) a government train carrying oxen

traveling through the northern plains of eastern Wyoming was caught in a snowstorm and had to be abandoned. The

driver returned the next spring to see what had become of his cargo. Instead of the skeletons he had expected to find,

he saw his oxen, living, fat, and healthy. How had they survived?

The answer lay in a resource that unknowing Americans lands trampled underfoot in their haste to cross the

“Great American Desert” to reach lands that sometimes proved barren. In the eastern parts of the United States, the

preferred grass for forage was a cultivated plant. It grew well with enough rain, then when cut and stored it would

cure and become nourishing hay for winter feed. But in the dry grazing lands of the West that familiar bluejoint grass

was often killed by drought. To raise cattle out there seemed risky or even hopeless.

Who could imagine a fairy-tale grass that required no rain and somehow made it possible for cattle to feed

themselves all winter? But the surprising western wild grasses did just that. They had wonderfully convenient features

that made them superior to the cultivated eastern grasses. Variously known as buffalo grass, grama grass, or

mesquite grass, not only were they immune to drought; but they were actually preserved by the lack of summer and

autumn rains. They were not juicy like the cultivated eastern grasses, but had short, hard stems. And they did not need

to be cured in a barn, but dried right where they grew on the ground. When they dried in this way, they remained

naturally sweet and nourishing through the winter. Cattle left outdoors to fend for themselves thrived on this hay. And

the cattle themselves helped plant the fresh grass year after year for they trampled the natural seeds firmly into the soil

to be watered by the melting snows of winter and the occasional rains of spring. The dry summer air cured them much

as storing in a barn cured the cultivated grasses.

101. What does the passage mainly discuss?

60

A. A type of wild vegetation B. Western migration after Civil War

C. The raising of cattle D. The climate of the Western United States

102. What can be inferred by the phrase “Legend has it” in line 1?

A. Most history book include the story of the train.

B. The story of the train is similar to other ones from that time period.

C. The driver of the train invented the story.

D. The story of the train may not be completed factual.

103. The word “they” in line 4 refers to _____.

A. plains B. skeletons C. oxen D. Americans

104. What can be inferred about the “Great American Desert” mentioned in line 7?

A. Many had settled there by the 1860’s.

B. It was not originally assumed to be a fertile area.

C. It was a popular place to raise cattle before the Civil War.

D. It was not discovered until the late 1800’s.

105. The word “barren” in line 7 is closed in meaning to _____.

A. lonely B. uncomfortable C. infertile D. dangerous

106. The word “preferred” in line 8 is closed in meaning to _____.

A. favored B. available C. ordinary D. required

107. Which of the following can be inferred about the cultivated grass mentioned in the second paragraph?

A. Cattle raised in the Western United States refused to eat it.

B. It had to be imported into the United States.

C. It would probably not grow in the western United States.

D. It was difficult for cattle to digest.

108. Which of the following was NOT one of the names given to the western grasses?

A. Mesquite grass B. Bluejoint grass C. Buffalo grass D. Grama grass

109. Which of the following was NOT mentioned as a characteristic of western grasses?

A. They contain little moisture B. They have tough stems

C. They can be grown indoors D. They are not affected by dry weather

110. According to the passage, the cattle help promote the growth of the wild grass by_____.

A. eating only small quantities of grass.

B. continually moving from one grazing area to another.

C. naturally fertilizing the soil.

D. stepping on and pressing the seeds into the ground.

IV. WRITING (6/20 points)

Part 1: Finish the second sentence in such a way that it means exactly the same as the sentence printed before it

111. Customs officials are stopping more travelers than usual this week.

→ An increased …………………………………………………………………………………………...

112. I left without saying goodbye as I didn’t want to disturb the meeting.

61

→ Rather ………………………………………………………………………………………………….

113. My decision to get up and dance coincided with the band’s decision to stop playing.

→ The moment …………………………………………………………………………………………...

114. He never suspected that the bicycle had been stolen.

→ At no time ……………………………………………………………………………………………...

115. How could I help, except to offer to lend her some money?

→ Other …………………………………………………………………………………………………...

Part 2: Use the word(s) given in the brackets and make any necessary additions to complete a new sentence in such

a way that it is as similar as possible in meaning to the original sentence. Do NOT change the form of the given

word(s).

116. To this day no one has equaled his achievements in the field of technology. (unsurpassed)

→ To this day ………………………………………………………… in the field of technology.

117. Peter grimaced as he swallowed the foul-tasting medicine. (pulled)

→ Peter ……………………………………………………………….the foul-tasting medicine.

118. It’s unfortunately that the construction of the building will not be finished as originally planned. (longer)

→ The construction of the building ………………………………………………………… unfortunate.

119. What he told me made me very curious to hear the rest of the history. (appetite)

→ What he told me ………………………………………………………………………….. the story.

120. They chose not to drive because they thought there would be too much snow. (fear)

→ They chose …………………………………………………………………….…. too much snow.

Part 3: Company ABC exports its products mainly to North America and Europe. The following chart shows the

sales figures of Company ABC from 2003 to 2008. Write a report (of about 150 words) to the company

management describing the information shows in the chart.

……………………………………………………………………………………………………………..

……………………………………………………………………………………………………………..

……………………………………………………………………………………………………………..

……………………………………………………………………………………………………………..

62

……………………………………………………………………………………………………………..

……………………………………………………………………………………………………………..

……………………………………………………………………………………………………………..

……………………………………………………………………………………………………………..

……………………………………………………………………………………………………………..

……………………………………………………………………………………………………………..

……………………………………………………………………………………………………………..

……………………………………………………………………………………………………………..

Part 4: Traffic jams are a common phenomenon in big cities in Vietnam. Write a report (of about 200 words) to the

municipal government providing suggestions as to how to solve the problem. Your report should include the

government and the citizens.

……………………………………………………………………………………………………………..

……………………………………………………………………………………………………………..

……………………………………………………………………………………………………………..

……………………………………………………………………………………………………………..

……………………………………………………………………………………………………………..

……………………………………………………………………………………………………………..

……………………………………………………………………………………………………………..

……………………………………………………………………………………………………………..

……………………………………………………………………………………………………………..

……………………………………………………………………………………………………………..

……………………………………………………………………………………………………………..

……………………………………………………………………………………………………………..

===== THE END =====

SỞ GD & ĐT NGHỆ AN

(Đề thi gồm 06 trang)

KỲ THI CHỌN HỌC SINH GIỎI TỈNH LỚP 12 THPT

NĂM HỌC 2012 – 2013

Môn thi: TIẾNG ANH 12 THPT - BẢNG A

Thời gian làm bài: 150 phút

SECTION A – PHONETICS

I. Choose the word whose underlined part is pronounced differently from that of the others.

1. A. breath B. break C. thread D. tread

2. A. believes B. pencils C. contents D. tables

3. A. ragged B. wicked C. naked D. packed

4. A. cherish B. chorus C. chaos D. scholar

63

Đề chính thức

5. A. hysteria B. hypocrite C. hypocrisy D. hydroplane

II. Find the word with the stress on the SECOND syllable.

6. A. comparable B. company C. comparative D. compass

7. A. committee B. refugee C. employee D. absentee

8. A. coherent B. permanent C. continent D. sentiment

9. A. fountain B. predict C. population D. colorful

10. A. unconcerned B. tropical C. represent D. conspicuous

SECTION B – VOCABULARY AND GRAMMAR

I. Choose the best answer from A, B, C or D to complete the following sentences.

11. She was ______ to discover that she had won the first prize.

A. excited B. lucky C. astonished D. nervous

12. ______ a minute! I can’t find my keys.

A. Keep on B. Hold on C. Go on D. Carry on

13. I have just taken a Test of English as a Foreign Language or TOEFL _______ short.

A. of B. in C. on D. for

14. She is a kind of woman who does not care much of work but generally _______ only with colleagues for meals,

movies or late nights at a club.

A. supposes B. socializes C. attends D. discusses

15. A good way of ______ food is keeping it in a fridge.

A. enduring B. extending C. prolonging D. preserving

16. Our seats were _____ far from the stage that we couldn't see the actors and actresses clearly.

A. very B. too C. enough D. so

17. It is very easy for the undereducated to be ______ by slick-talking salesmen.

A. put aside B. put up C. taken in D. taken away

18. The ______ of the family following the divorce was a great shock to the children.

A. break-down B. break-in C. break-up D. break-out

19. I can’t believe how cheap these shorts were. They were a real ______.

A. discount B. sale C. bargain D. offer

20. Never ______ off until tomorrow what you can do today.

A. set B. put C. do D. turn

21. I have got a _____ headache.

A. spitting B. raving C. splitting D. burning

22. For a while I was at a ______ to know what to say.

A. blank B. loss C. pain D. crisis

23. I don’t think this fashion will ______.

A. catch on B. catch up C. catch out D. catch over

24. We need guaranteed financial ______ before we can even start the design work.

64

A. agreement B. backing C. analysis D. plans

25. The telephone rang and interrupted my ______ of thought.

A. train B. chain C. series D. circle

II. Read the following passage. There is ONE mistake in each of the numbered lines. Find and correct it.

26

27

28

29

30

The UK Government ensures that all schools in the UK meet certain standards, and this includes

independent schools as well as those are run by the Government. All qualifications are awarded by

national agencies accredited by the Qualification and Curriculum Authority (QCA), so that the quality of

the qualifications you will gain is guaranteed.

At many independent schools in England, you will encourage to take part in extracurricular activities to develop

your hobbies and learn new skills, and you may be encouraged to take graded music exams offering by the

Associated Board of the Royal Schools of Music, or Trinity College. The exam grades gained from these are

wide accepted toward university entry requirements.

Mistake Correction Mistake Correction

26 29

27 30

28

III. Supply the correct forms of the words in brackets.

31. The judges were especially impressed by the (IMAGINE) ______ use of light and shade in the painting.

32. Slavery is the opposite of (FREE) ______.

33. The athlete (AMBITION) ______ decided to aim for three gold medals.

34. The villages in the mountains are quite (ACCESS) ______ during winter.

35. In critical situations, they often lose because they play so (PROFESSION) ______.

SECTION C – READING

I. Read the following passage, and then choose the best answer from A, B, C or D.

WHY SEAT BELTS ARE NECESSARY

More than 30,000 drivers and front seat passengers are killed or seriously injured each year. The impact on you of

an accident can be very serious. At a speed of only 30 miles per hour it is the same as falling from a third - floor

window. Wearing a seat belt saves lives; it reduces your chance of death or serious injury by more than half.

Who has to wear a seat belt?

Drivers or front seat passengers in most vehicles. If you are 14 or over it will be your responsibility to wear the

belt. If you do not, you could be fined up to £50. It will not be up to the driver to make sure you wear your belt. But it

will be the driver’s responsibility to make sure that children under 14 do not ride in the front unless they are wearing a

seat belt of some kind.

65

A very few vehicles have a middle front seat between the front passenger seat and the driver’s seat, for example a

bench seat. Your vehicle may be one of them. If just one passenger sits in front, he must wear a seat belt. But if two

passengers sit in front, the person sitting in the middle will not have to wear a belt.

When you do not have to wear a seat belt

You do not have to wear a seat belt in certain circumstances, such as if you are reversing your vehicle, if you are

making a local delivery or collection using a vehicle constructed or adapted for that purpose, or if you have a valid

medical certificate which excuses you from wearing it. Make sure these circumstances apply to you before you decide

not to wear your seat belt. Remember you may be taken to court for not doing so, and you may be fined if you cannot

prove to the court that you have been excused from wearing it.

Medical exemptions

Certain people ought not to wear a seat belt because of their health. It may be more risky for them to wear a belt

than to be in a road accident without one. But they will not have to wear a belt if they get a valid medical certificate

from a doctor. If you think this applies to you, go and talk to a doctor as soon as possible. The doctor may reassure

you that you can wear a seat belt. Or he may have to examine you before he can decide whether or not to give you a

certificate. When you go and see him you should ask him at the start how much this would cost. Keep the certificate.

If the police ask you why you are not wearing a seat belt, you should show them the certificate. If you cannot show it

to them on the spot, you should take the certificate to a police station of your choice within five days.

36. This text is taken from ______.

A. a medical magazine B. a police report on safety

C. a legal document D. a government information leaflet

37. Wearing a seat belt in a vehicle ______.

A. reduces the risk of death and injury to drivers and passengers

B. reduces road accidents by more than half

C. saves lives only at a speed of 30 miles per hour

D. saves the lives of more than 30,000 drivers and front seat passengers

38. It is the driver’s responsibility to ______.

A. make the front seat passenger wear a seat belt

B. stop children riding in the front seat

C. wear a seat belt on all occasions

D. make children under 14 wear a seat belt in the front

39. According to the text, which of the following people does not have to wear a seat belt?

A. Someone who is picking up the children from the local school.

B. Someone who is backing into a parking space.

C. Someone who is delivering invitations to a party.

D. Someone who is under 14.

66

40. If you are excused from wearing a seat belt on medical grounds, ______.

A. you must take the certificate to the police station within five days

B. the doctor will give you a certificate

C. you must show the certificate to the police on the spot

D. the doctor will have to examine you

II. Choose the best answer from A, B, C or D to fill in the gaps in the following passage.

MUSIC - A UNIVERSAL LANGUAGE

Music is universal - it is produced by all cultures. Some scientists believe that music came before speech

and (41) ______ as a development of mating calls. In fact, there is one theory that the (42) ______ languages

were chanted or sung, rather than spoken. Indeed, in some cultures, music is a form of (43) ______ history. The

Aboriginal Australians, for example, use music as a means to (44) ______ on stories of the land and spirits to the next

generation.

New evidence suggests that music does not just (45) ______ the feel - good factor but it is also good for the

brain. A study of intellectually (46) ______ children showed that they could recall more information after it was given

to them in a song than after it was read to them as a story.

Researchers also report that people (47) ______ better on a standard intelligence test after listening

to Mozart. The so-called “Mozart effect” has also been (48) ______ by findings that rats brought up on

Mozart run faster through a complex network of paths or passages, (49) ______ as a maze. Overall, it seems

that in most instances people who suffer from any form of mental (50) ______ benefit from listening to music.

41. A. was B. swelled C. arose D. reacted

42. A. earliest B. newest C. easiest D. simplest

43. A. enjoying B. making C. recording D. stating

44. A. move B. pass C. hand D. happen

45. A. convince B. satisfy C. please D. prefer

46. A. disabled B. inactive C. incapable D. disordered

47. A. examine B. prepare C. achieve D. score

48. A. supported B. given C. marked D. remembered

49. A. called B. heard C. regarded D. known

50. A. badness B. hurt C. illness D. pain

III. Fill in each numbered gap with ONE suitable word.

FRUITIBIX

Do you want to be slim?

Do you worry about your family’s health?

Then you should try Fruitibix, the new (51) ______ fruit and nut biscuit. Fruitibix (52) ______

wonderful, but it contains less sugar than most other biscuits. Each biscuit contains dried fruit and nuts,

67

(53) ______ apples, coconut and bananas. Sometimes you (54) ______ like eating something between

meals. Now, instead of (55) ______ a chocolate bar, bite into a Fruitibix. It will keep you healthy.

At lunchtime, instead of chips or hamburgers, have a Fruitibix. It contains all the essential foods for a balanced

(56) ______. And if you are in a (57) ______, and don’t have time for a proper meal, Fruitibix will give you the (58)

______ to keep on going.

So whenever your children ask for (59) ______ sweet, give them Fruitibix instead of cakes or chocolate. They

will love the taste and it won’t (60) ______ their teeth. Discover Fruitibix. It’s on your supermarket shelves now!

SECTION D – WRITING

I. Complete the second sentence so that it has the same meaning as the first one.

61. I can't believe this is the best hotel in the city! 

→ There must ..............................................................................................................................

62. Your scheme is brilliant, but I do not think it will work.

→ Brilliant ..................................................................................................................................

63. The two sides never looked likely to reach an agreement.

→ At ............................................................................................................................................

64. We must continue our efforts whether there are problems or not.

→ Regardless ...............................................................................................................................

65. Basically, a couple’s happiness depends on their ability to communicate.

→ Basically, the ..........................................................................................................................

II. Use the word given in brackets and make any necessary additions to write a new sentence in such a way that it

is as similar as possible in meaning to the original sentence. Do NOT change the form of these words.

66. If you are in London by any chance, come and see me. (HAPPEN)

→ ................................................................................................................................................

67. Fred tried hard to start the car, but without success. (MATTER)

→ ................................................................................................................................................

68. I didn’t agree with the idea. (FAVOR)

→ ................................................................................................................................................

69. He pretended to be enjoying himself, but he wasn’t really. (AS)

→ ................................................................................................................................................

70. He is certainly not a reliable witness. (MEANS)

→ ................................................................................................................................................

III. Write a composition about 150 – 200 words on the following topic:

“PARENTS ARE THE BEST TEACHERS”.

Do you agree or disagree with this statement? Use specific reasons and examples to support your answer.

SỞ GD & ĐT NGHỆ AN KỲ THI CHỌN HỌC SINH GIỎI TỈNH LỚP 12

68

NĂM HỌC 2012 - 2013

Môn thi: TIẾNG ANH LỚP 12 THPT - BẢNG B

Thời gian làm bài: 150 phút

SECTION A – PHONETICS

I. Choose the word whose underlined part is pronounced differently from that of the others.

1. A. challenge B. chance C. achievement D. scholarship

2. A. headaches B. wishes C. finishes D. watches

3. A. parked B. laughed C. ploughed D. established

4. A. musician B. sociable C. society D. official

5. A. flew B. few C. sewage D. drew

II. Identify the word whose stressed pattern is different from that of the others.

6. A. intelligent B. population C. opportunity D. economics

7. A. advisable B. admirable C. reliable D. desirable

8. A. method B. invention C. intense D. effective

9. A. technology B. important C. popularity D. impossible

10. A. solemn B. express C. succeed D. event

Section B – vocabulary and grammar

I. Choose the best answer from A, B, C or D.

11. You are old enough to take _______ for what you have done.

A. responsible B. responsibility C. responsibly D. irresponsible

12. Should gravity, the pull of the earth, be zero, everything ____________ .

A. will float B. would be floating C. floated D. would float

13. I’ll have to work hard to _______ with the rest of the class.

A. catch B. run C. keep pace D. keep on

14. _______ his exhaustion, he won the marathon by nearly three minutes.

A. In spite B. Despite C. Although D. However

15. Your success in life very largely on yourself.

A. is keen B. depends C. lies D. goes

16. Buying a car was an important __________ for them.

A. transformation B. translation C. transaction D. transportation

17. No one can predict the future exactly. Things may happen _______.

A. expected B. unexpected C. expectedly D. unexpectedly

18. _______! The tree is going to fall.

A. Look out B. Look up C. Look on D. Look after

19. We took _____ of the fine weather and spent the day on the beach.

69

Đề chính thức

A. chance B. advantage C. occasion D. effect

20. All the athletes taking part in the international games should be proud of ________ .

A. oneself B. themselves C. himself D. yourself

21. The bomb _______ in the garage; fortunately no one hurt.

A. put on B. went off C. got out D. kept up

22. __________ I can’t understand is _________ he wants to change his mind.

A. That/that B. Which/what C. What/why D. What/that

23. The nurse was on _____ in the hospital all night.

A. duty B. alarm C. service D. work

24. Only when you grow up ______ the truth.

A. you will know B. you know C. do you know D. will you know

25. All the boys are good at cooking, but _______ is as good as the girls.

A. none B. either C. neither D. every

II. There is one mistake in each of the following sentences. Find and correct it.

26. Mrs. Stevens, along with her cousins from New Mexico, are planning to attend the festivities.

27. Some of us have to study their lessons carefully if we expect to pass this examination.

28. A five-thousand-dollars reward was offered for the capture of the escaped criminals.

29. Many kinds of vegetables are growth in California’s Imperial Valley.

30. Despite the metric system is used throughout the world, it is still not commonly used in the Unite States.

31. She is looking forward to meet him again.

32. In 1837 Victoria, an eighteen-year-old woman, named queen of England.

33. The tickets that you ordered they will be delivered tomorrow.

34. Cool temperatures, shade, moist, and the presence of dead organic material provide the ideal living conditions for

mushrooms.

35. When I first went to a university I studied law.

Mistake Correction Mistake Correction

26. 31.

27. 32.

28. 33.

29. 34.

30. 35.

Section C – reading

I. Read the following passage, and then choose the best answer from A, B, C or D.

When you enter the supermarkets, you see shelves full of food. You walk in the aisles between the shelves. You

push a shopping cart and put your food in it.

70

You probably hear soft, slow music as you walk along the aisles. If you hear fast music, you walk quickly.

The supermarket plays slow music, you walk slowly and have more time to buy things.

Maybe you go to the meat department first. There is some meat on sale, and you want to find it. The manager

of the supermarket knows where customers enter the meat department. The cheaper meat is at the other end of the

meat department, away from where the customers enter. You have to walk by all the expensive meat before you find

the cheaper meat. Maybe you will buy some of the expensive meat instead of the meat on sale.

The dairy department sells milk and milk products such as butter and cheese. Many customers like milk that

has only little butterfat in it. One store has three different containers of low fat milk. One says ‘1% fat’ on the

container. The second says ‘99 percent (99%) fat free’. The third says ‘Low fat’ in big letters and 1% in small letters.

As you can see, all the milk has the same amount of fat. The milk is all the same. The amount of milk in each

container is also the same. However, in this store they cost three different amounts of money. Maybe the customers

will buy the milk that costs the most.

36. What is the main topic of this passage?

A. How different kinds of food are arranged in supermarkets.

B. Soft, slow music makes people buy more in supermarkets.

C. The supermarket is designed to make you buy things.

D. Be sure not to be deceived in supermarkets

37. The manager knows __________.

A. which customers like low fat milk B. which customers like slow music

C. where customers enter the meat department D. where customers come from

38. When you walk by the expensive meat __________ .

A. maybe you will buy some B. maybe you will buy low fat milk

C. you will look for fresh food D. you will walk on the shelves

39. The word “they” in the last paragraph means ______.

A. the customers B. the managers of the supermarket C.

the containers of low fat milk D. the food in the supermarket

40. There are three different containers of low fat milk.

A. One has 99 percent of butterfat.

B. They all cost the same amount of money.

C. One has less fat than the others.

D. They all have the same amount of fat.

II. Choose the best answer from A, B, C or D to fill in the gaps in the following passage.

British parents are always complaining that their children spend too much time glued to the telly and not

enough time on other activities (41)_________ sports and reading. A survey recently (42)_________ on people’s

viewing habits does not disprove this. It shows that young people in Britain spend on average 23 hours a week in front

of the television, (43)_________ works out at over three hours every day.

71

(44)__________ is surprising, however, is the fact that the average adult watches even more: incredible 28 hours

a week. We (45)__________ to have become a nation of addicts. Just about every household in the country has a

television and over half have two or more. According to the survey, people nowadays don’t just watch television

sitting in their living-rooms, they watch it in the kitchen and in bed (46)_________.

The Education Minister said a few weeks ago that Britain’s pupils (47)_________ spend more time reading.

Unfortunately, parents are not setting a good example: adults do (48)_________ reading than young people. In fact,

reading is at the (49)_________ of their list of favourite pastimes. They would (50)_________ listen to the radio, go

to the cinema or hire a video to watch on their televisions at home.

41. A. such B. like C. as D. alike

42. A. investigated B. researched C. carried D. carried out

43. A. that B. which C. this D. it

44. A. What B. It C. The thing D. This

45. A. seem B. ought C. used D. would like

46. A. in addition B. as well C. more D. moreover

47. A. might B. could C. should D. would

48. A. more B. less C. little D. fewer

49. A. tail B. top C. beginning D. bottom

50. A. better B. rather C. prefer D. like

III. Fill in each numbered gap with ONE suitable word.

The traditional of birthday parties started a long time ago. People thought that there (51)______ a special

danger from evil spirits, so friends and family came together to bring good thoughts and wishes and even presents. At

one time, only kings had birthday parties but as time went (52)____, children and then adults began to have their

(53)_____ birthday celebrations. There are some traditions - (54)______ as sending birthday cards, blowing out the

candles on a birthday cake and singing the ‘Happy Birthday’ song, that you can find (55)_______ anywhere, any time.

In China, everyone celebrates their birthday on New Year’s Day: they become one year (56)______ on that

day. On a child’s second birthday, family members put a variety of objects on the floor around the child. (57)______

to Chinese tradition, the first object that the child picks up tells you what profession the child will choose (58) ______

in life.

In many English-speaking countries, a twenty-first birthday cake often has a key on top, or the cake

(59)______ is sometimes in the shape of a key. The key means that the young person is now old (60)______ to leave

and enter the family home at any time they want to!

Section D – writing

I. Complete the second sentence so that it has the same meaning as the first one.

61. The heavy rain prevented everybody from going out.

→ Had __________________________________________________________________________________

62. Mrs. Taylor regretted buying the second-hand washing machine.

72

→ Mrs. Taylor wished______________________________________________________________________

63. They believe that the wanted man was living in London.

→ The wanted man ________________________________________________________________________

64. What is the weight of your suitcase?

→ How__________________________________________________________________________________

65. In spite of his age, Mr. Benson runs 8 miles before breakfast everyday.

→ Though_______________________________________________________________________________

II. Use the word given in brackets and make any necessary additions to write a new sentence in such a way that it

is as similar as possible in meaning to the original sentence. Do NOT change the form of these words.

66. If I met the author one day, I would ask him to sign my copy of his new book. (Were)

→ _____________________________________________________________________________________

67. “It was your fault to break my vase yesterday” said Jane to her brother. (accused)

→ _____________________________________________________________________________________

68. She is such a beautiful girl that everybody admires her. (so)

→ _____________________________________________________________________________________

69. Are you and your husband the same age? (old)

→ ______________________________________________________________________________________

70. Hoang didn’t come to class today. Nhan didn’t, either. (nor)

→ _____________________________________________________________________________________

III. Write a composition about 150 – 200 words on the following topic:

“Students learn just because of good marks”. Do you agree or disagree with that statement?

SỞ GD&ĐT NINH BÌNH ĐỀ THI CHỌN HỌC SINH GIỎI LỚP 12 THPT

Kỳ thi thứ hai - Năm học 2012 – 2013

Ngày thi 18/12/2012

(Thời gian 180 phút không kể thời gian giao đề)

PART B. LEXICO-GRAMMAR (8 points)

I. Choose the word or phrase (A, B, C or D) which best completes each sentence. Write your answer

in the numbered box.

11. We have a _______ future ahead with little comfort, food or hope.

A. cruel B. pessimistic C. grim D. fierce

12. Don’t be late for the interview, _______ people will think you are a disorganized person.

A. unless B. otherwise C. if not D. or so

13. ______ the rise in unemployment, people still seem to be spending more.

A. Nevertheless B. Meanwhile C. Despite D. Although

73

ĐỀ THI CHÍNH THỨC

14. This meat is rather tough; you must chew it well before _______

A. swallow B. to swallow C. swallowed D. swallowing

15. The police have warned the tourists to look ________ for pickpockets in the shopping centre.

A. away B. down C. out D. forward.

16. At the language school, each student is assigned to his or her own ________.

A. tutor B. professor C. staff D. director

17. Many _______ are in danger of extinction.

A. plants B. animals C. species D. hippopotamus

18. Do you ________ my turning the television on now?

A. disapprove B. mind C. want D. object

19. As soon as you buy a car, it starts falling in ________.

A. cost B. worth C. price D. value

20. I saw a thief take Norman’s wallet so I ran ________ him, but I couldn’t catch him.

A. after B. into C. over D. near

II. Put the verb in each bracket into correct tense. Write the answer in the numbered box.

This time last year I (21. cycle) ___________in the rain along a country road in France with a friend

of mine. We (22. decide) _________to go on a cycling holiday in Normandy. Neither of us had gone to

France before, but we (23. know) ___________ some French from our time at school and we had

managed to rush up on the basics. We (24. plan) _______ our route carefully in advance, but we had

forgotten one important thing, the weather. It (25. rain)_______ solidly since our arrival and that night we

(26. end up) _________ sleeping in the waiting room at a railway station. Then the next morning as we

(27. ride) _______down a steep hill my bike skidded on the wet road and I (28. fall off)_________.I

realised immediately that my arm (29. break) ___________, and after a visit to the local hospital I caught

the next train to Calais for the ferry home. Unfortunately, my parents (30. not expect) _______ me home

for a fortnight, and had gone away on holiday. So I spent a miserable couple of weeks alone, reading

Teach Yourself French.

III. Give the correct form of the word in each bracket to best complete each sentence. Write the

answer in the numbered box.

31. My teacher is very ______________ about the history of Vietnam. (KNOW)

32. I’m sorry for my ______________, but I hate being kept waiting. (PATIENT)

33. The American celebrate their ___________ on 4th July. (DEPEND)

34. She always listens ___________ to what she is told. (ATTENTION)

35. We always have a bed ready in the spare room in case visitors arrive _____. (EXPECT)

36. George is ________. He always plays with toys. (CHILD)

37. Where is ________ to this shopping center? (ENTER)

38. Do you often wear ________ costume in the New Year Celebration? (NATION)

39. She was ________ keen on the history of China. (EXTREME)

40. She studied ________ at university. (ECONOMY)

74

IV. Each sentence below has ONE mistake. Underline and correct it. Write your answer in the right

column. 0 is an example.

Sentences Correction

0. I am studying here since last January

41. As soon as I will graduate, I’m going to return to my hometown.

42. The more than sugar a person eats, the less likely he is to lose weight.

43. By the end of the 21th century, scientists will had discovered the cure for the

common cold.

44. Aspirin is recommend to many patients for its effect to thin the blood.

45. Beethoven wrote and performance some of his greatest works while he

was almost totally deaf.

46. She is no longer enough young to take part in a beauty contest.

47. Thirty hours a week are a heavy work schedule.

48. The governor sent the National Guard to assist the local habitants in the

clean – up operation because the torrential rain that had devastated the area.

49. Some studies show that young babies prefer the smell of milk to those of

other liquids.

50. Financial considerations play an important part in the choose of a college.

have been

studying

41. ____________

42. ____________

43. ____________

44. ____________

45. ____________

46. ____________

47. ____________

48. ____________

49. ____________

50. ____________

PART C. READING (6 points)

I. Read the following passage and decide which answer (A, B, C, or D) best fits each gap.

Mr. Brown lived his entire life in a small town in the north of England. He never left the house

___________(51) he had been born, never married, never went on holiday and had no friends. He worked

in a local factory for _________(52) forty years but even the people who had worked with him for years

_________(53) very little about him. He wore the same clothes year in year out, and _________(54) he

shopped regularly at the local store he bought only the most basic foodstuffs, never changing his

purchases from one week to the next. So when he died last month, neighbors and local people were

_________(55) to learn that Mr. Brown was not just a rich man, he was in fact __________(56) millionaire!

He had no bank, no account, no money invested anywhere but in the various drawers, cupboards

and boxes in the house there were hundreds and thousands of bank _________(57) and coins.

It took police over two weeks to clear the house and the bank clerks took just as long to

___________(58) all the money. “We had absolutely no ideas that he had been hiding his money over the

years”, one of his neighbors said. “In fact, we used to feel _________(59) for him, we thought he was a

poor old man unable to __________(60) anything better for himself”.

51. A. which B. where C. who D. what

52. A. more B. above C. over than D. more than

53. A. reminded B. held C. had D. knew

54. A. although B. but C. because D. however

75

55. A. astonishing B. astonished C. interested D. interesting

56. A. the B. this C. a D. one

57. A. papers B. money C. letters D. notes

58. A. number B. count C. guess D. make

59. A. sorry B. sad C. sorrow D. miserable

60. A. pay B. spend C. afford D. give

II. Read the passage carefully and fill in each of the blanks with ONE suitable word.

When we talk about the environment today, many people still think only of tree ________(61) and

keeping their surroundings clean. Although both activities are _________(62), they are only a small part of

the whole story.

The environment is __________(63) urban and rural. It ___________(64) the seashore, rivers,

mountains, cities, towns, villages and so on. The water we drink and the air we ________(65) are important

elements of the environment, so __________(66) the trees, plants and animals. The environment

___________(67) us with all our daily needs. This is true for both the urban and rural people. Rural people

________(68) directly on the natural environment for many of their daily needs. They use water from rivers,

streams and so on. For them, there is often no piped water available. They collect firewood from nearby

trees and forests for cooking and heating. They _________(69) the land around them for food. Their

homes are _________ (70) from local materials.

III. Read the passage carefully and answer the questions that follow.

Many people wrongly believe that when people reach old age, the families place them in nursing

homes. They are left there in the hands of strangers for the rest of their lives. Their grown children visit

them only occasionally. The truth is that this idea is an unfortunate myth – a fictitious story. In fact, family

members provide over 80 percent of care that elderly people need.

Because Americans are living longer than ever, more psychologists and social workers have begun

to study care giving to improve care of the elderly. They have found that all caregivers share a common

characteristic: All caregivers believe they are the best person for the job. One caregiver said that she had

always been closed to her mother. Another was the oldest child. Another was the youngest child. Social

workers interviewed caregivers to find out why they took on the responsibilities of caring for an elderly,

dependent relative. They discovered three basic reasons. Many caregivers believed that they had an

obligation to help their relative. Some stated that helping others made them more useful. Others hoped that

by helping someone now, they would deserve care when they became old and dependent.

When people care for an elderly relative, they often do not use available community services, such

as adult daycare centers. If the caregivers are adult children, they are more likely to use such services,

especially because they often have jobs and other responsibilities. In contrast, a spouse, usually the wife,

is much less likely to use support services or to put the dependent person in a nursing home. Social

workers discovered that the reason for this difference was fear of poverty. All ill, elderly people may live

years, and medical care and nursing homes are very expensive. An elderly couple’s saving can disappear

76

very quickly. The surviving spouse, usually the wife, can be left poverty. As a result, she often tries to take

care of her husband herself for as long as she can.

Researchers have found that caring for the elderly can be a very positive experience. The elderly

appreciated the care and attention they received. They were affectionate and cooperative. However, even

when care giving is satisfying, it is hard work. Social workers and experts on aging offer caregivers and

potential caregivers help when arranging for the care of an elderly relative. One consideration is to ask

parents what they want before they become sick or dependent. Perhaps they prefer going into a nursing

homes and can select one in advance. On the other hand, they may want to live with their adult children.

Caregivers must also learn to be assertive and ask for help from others, especially siblings. Brothers and

sisters are willing to help, but they do not know what to do.

We can expect to live longer lives than ever before. Caring for the elderly and being taken care of

can be a mutual satisfying experience for everyone involved.

Questions 71-74: Read the passage and decide whether the following statements are true or false.

Write True/False in the numbered box.

71. Most of old people in the United States live in the care of their families.

72. All caregivers believe they can do the job better than anyone else.

73. There are at least three reasons why caregivers take on responsibilities for an elderly person.

74. Care giving is not too difficult when the elderly are affectionate and cooperative.

Question 75-77: Answer the following questions.

75. What is the unfortunate myth about care of the elderly in the United State?

…………………………………........................................…………………………………………………..

…………………………………........................................…………………………………………………..

76. Why are adult children more likely to use community services to help care for an elderly parent?

………………………………………………......................................………………………………………

77. What advice do social workers give to people when they arrange for the care of elderly relative?

………………………………………………......................................………………………………………

Questions 78 - 80: Choose the best answer A, B or C for the following questions.

78. What is the main idea of this passage?

A. Most people are put into nursing homes by their families, who do not visit them regularly.

B. Most old people live longer today than they did in the past.

C. Most elderly people are taken care of by their families, who often find the experience satisfying.

79. What does “in the hands of” in paragraph 1 mean?

A. In the control of B. In the care of C. In the homes of

80. What does “siblings” in paragraph 4 mean?

A. Care givers B. Brothers and sisters C. Mothers and fathers

PART D. WRITING (4 points)

I. Arrange the given words to make meaningful sentences. 0 is an example.

77

0. yet/ The/ finished/ football/ players/ training/ haven’t/ their.

The football players haven’t finished their training yet.

81. how/ People/ would/ used/ to/ know/ or/ less/ their/ more/ children/ live.

…………………………………………………………………………………………………………………

82. find/ do/ this/ you/ interesting/ What/ about/ city?

…………………………………………………………………………………………………………………

83. at/ If/ only/ I/ studied/ had/ harder/ school.

…………………………………………………………………………………………………………………

84. last/ because/ stayed/ up/ You/ are/ you/ tired/ late/ night.

…………………………………………………………………………………………………………………

85. non-professionals/The/ star/ hates/ be/ film/ to/ criticized/ by.

…………………………………………………………………………………………………………………

II. Finish each of the following sentences in such a way that it means exactly the same as the

sentence printed before it.

86. 'Shall I carry that bag for you, John?' said Pauline.

Pauline offered………………………………………………………………………………

87. I’ve forgotten that author’s name but he’s very well-known.

That author,..............................................................................................................................

88. It’s extremely difficult for us to make ends meet these days.

We find…….………………………………………………………………………….……..

89. She prefers reading a book to watching TV.

She would .……………………………..……………………………...................................

90. Alice and Charles didn’t decide to move to a bigger house until after the birth of their second child.

Only when………………………………………………………………………………

III. Many people think that: “Television does harm to children.” Do you agree or disagree with their

idea? In about 160 words, write a paragraph to support your viewpoint.

SỞ GIÁO DỤC VÀ ĐÀO TẠO TUYÊN

QUANG

KỲ THI CHỌN HỌC SINH GIỎI

CẤP TỈNH LỚP 12 THPT NĂM HỌC 2010-2011

SECTION II: LEXICO-GRAMMAR (9 points)

Part 1: Choose ONE word or phrase which best completes each sentence. Circle its

corresponding letter A, B, C, or D to indicate your answer. (4 points)

1. They _________ him of having stolen the car.

A. blamed B. accused C. complained D. convicted

2. I was wearing a heavy raincoat to _________ myself from the cold.

A. conceal B. hide C. protect D. wrap

3. The new discovery was a major _________ for research workers.

A. breakthrough B. break-down C. breakout D. break-in

78

4. The film lasted every three hours with _________ of 15 minutes between part one and part two.

A. a pause B. an interruption C. an interval D. a stop

5. The nurse _________ in the hospital helped the patient come round.

A. on alarm B. on duty C. in service D. in shift

6. Buses are not very _________. Sometimes, they come every five minutes, then other times you have to wait for

half an hour.

A. in time B. on time C. punctual D. timely

7. The _________ is the official who makes sure the rules are followed in tennis.

A. umpire B. instructor C. referee D. guide

8. Do you have a non-smoking room with two beds _________ for next Friday and Saturday?

A. empty B. available C. vacant D. ready

9. Workers demanded pay rises to take account of the _________ of inflation.

A. percent B. percentage C. estimate D. rate

10. After his business failed, Mr. Johnson was declared _________ by the court.

A. penniless B. profitless C. bankrupt D. lost

11. Don’t _________ ! I haven’t finished explaining yet!

A. hang up B. hold on C. hang on D. call up

12. It _________ out that the mayor had bribed several councilors to vote.

A. resulted B. pointed C. broke D. turned

13. It’s difficult to tell identical twins_________.

A. on B. out C. apart D. over

14. No sooner had we started the picnic _________ the rain began pouring down!

A. than B. when C. that D. and

15. _________ you try, you can never get them all right.

A. How hard B. However hard C. For as hard as D. So hard as

16. He had an excellent grade in his examination _________ the fact that he hadn't worked particularly hard.

A. in spite of B. although C. because of D. on account of

17. The child was told to _________ for being rude to his uncle.

A. excuse B. apologize C. confess D. pardon

18. She _________ her neighbor’s children for the broken window.

A. blamed B. accused C. complained D. convicted

19. Both drivers were _________ in the accident and were taken to hospital.

A. damaged B. beaten C. injured D. wounded

20. Mary’s broken arm was so _________ that she burst into tears.

A. aching B. painful C. sharp D. hurt

Part 2: Supply the correct form of verbs in the blank. (2 points)

1. After 3 days of rain, I’m glad that the sun (shine) _______________ again.

2. I (make) _______________ cakes. That’s why my hands are all covered with flour.

79

3. When you (get) _______________ married? – Next Sunday.

4. He (break) _______________ his leg yesterday as he (repair) _______________ the roof.

5. My sister (lose) _______________ her hat and she (look) _______________ for it until now.

6. Faternah looked down to discover a snake at her feet. When she saw it, she (scream) ___________.

7. By the time Peter finally graduated from school, he (attend) _______________ seven different schools because his

parents moved frequently.

8. Until you learn to relax more, you (not improve) _______________ your ability to speak English.

Part 3. Supply the correct form of the word to fill in the blank. (2 points)

Healthy eating is (1) ________ the key to general well-being. Our bodies are made

up of what we eat, so our (2) ________ cannot possibly escape the effects of bad

diet and neither can our (3) ________ . Sweets, chocolate and cake are fine in (4)

________ , but trouble arises when people just cannot leave them alone, (5)

________ eating every possibly sticky item that comes their way. (6) ________ is

available for serious problems but (7) ________ is normally better than cure.

Make a careful (8) ________ when it comes to desserts, and favour cafes that

offer a good (9) ________ of fruit to round off a meal. A (10)________ in your

sugar intake may well hurt at first but you will feel better for it.

DOUBT

FIT

VITAL

MODERATE

GREED

TREAT

AVOID

CHOOSE

SELECT

REDUCE

Part 4: Circle an underlined word or phrase that needs correction. (1 point)

1. He looks remarkable like his father.

A B C D

2. The bridge was hitting by a large ship during a sudden storm last week.

A B C D

3. What does she often have for the lunch?

A B C D

4. The director doesn’t allow smoke in this office.

A B C D

5. In the early days of their development, cars used a great numbers of fuel.

A B C D

SECTION III. READING COMPREHENSION (4,4 points)

Part 1: Read the following passage then decide which is the best choice to the statement. Identify your answer

by circling the corresponding letter A, B, C or D. (1,4 points)

The Garbage Problem

80

Garbage is a big problem all over the world. People buy and use a lot of things nowadays. After a while, they

throw them away in the garbage bin. All the garbage is later thrown away or dumped outside the city. These places

are called landfill sites. In many cities, landfill sites are now full.

About one third of all the garbage is made of paper. Another third of the garbage is a mix of glass, metal,

plastic, and wood. The final third comes from food scraps. These are remains of food that are now not eaten anymore.

Food scraps are not a big garbage problem for the environment. Our natural world can get rid of food scraps. Insects

and bacteria eat food scraps and make them go away.

But this does not happen with other materials. Plastic is very toxic to the environment. It poisons the earth and

the water. We use plastic for many things such as combs or pens. Also, when we buy something from the

supermarket, we get a plastic bag. As soon as we get home, we throw the bag away. Plastic is also used to make

Styrofoam. All take-out coffee cups and fast-food are made of Styrofoam. When we buy coffee and drink it on the

street, we throw that cup away too.

Other garbage we throw away is metal. The cans for soft drinks or beer are made of aluminum. Aluminum is

toxic too. The paper and wood we throw away are not toxic. But we have to cut down many trees every year to make

paper and wood. Our environment suffers when there are no forests around. The air is less fresh, and the earth dries

up. With no water in the earth, plants cannot grow.

Solutions to the garbage problem

We have to manage our waste and garbage better. If we throw away so many things, soon we will have no place

to dump them.

The best thing to do is to reduce the amount of garbage. If we use less, we throw away less. For instance, we

can buy food in big boxes and packages. Then we throw away only one box every month or so. Otherwise, we throw

away many small boxes or cans every day.

Similarly, we can reuse a lot of packaging. For example, we do not have to buy take-out coffee in Styrofoam

cups. We can bring our own cup from home and fill it with fresh coffee. We also do not have to take the plastic bags

from the supermarket. We can bring our own cloth bag from home instead.

When we pack lunch, it is better to use a lunch box than a paper bag. Instead of paper plates, we can use real

plates. We can clean up with a dishtowel, not a paper towel. We can use a compost bin for food scraps. In this way,

the food gets back into the earth. It does not get mixed up with the regular garbage.

Finally, all paper, glass and metal we do use, we can recycle. In many countries, there are now recycling

programs. In Germany, for example, people separate all glass bottles by color. Then they put the bottles into special

bins that are on the street. The city collects the glass, cleans it, and reuses it. As well, in most countries, people recycle

newspapers and cardboard. It is easy and efficient.

1. What is the topic word of the first paragraph?

A. Throw away B. Garbage C. Plastic D. Environment

2. What is the main idea of the second passage?

A. People must deal with garbage better. B. People should reuse things.

C. People should recycle more. D. People should reduce their waste.

3. Why does the author say that garbage is a big problem?

81

A. Because people buy too many things.

B. Because people throw away everything they buy.

C. Because not all cities have landfill sites.

D. Because landfill sites get fewer and fewer.

4. What do people throw away?

A. Paper and wood B. Plastic, glass and metal.

C. Food scraps. D. All of the above.

5. Why does the author mention Germany at the end of the reading passage?

A. To suggest that recycling is the best solution to the garbage problem.

B. To offer additional advice about how to handle waste.

C. To criticize countries that do not have a recycling program.

D. To demonstrate that recycling works.

6. Why should people not throw away Styrofoam cups?

A. Because they are toxic to the environment.

B. Because they can reuse them again at home.

C. Because they can buy take-out coffee in them.

D. Because they can fill them again with fresh coffee.

7. What advice does the author give about reusing waste?

A. We should drink take-out coffee.

B. We should use our own bags, cups and plates.

C. We should throw away food into the landfill sites.

D. We should clean glasses ourselves and recycle them.

Part 2: Read the passage and choose one word or phrase that best fits each space (3 points)

What kind of music do you (1) ________ ? Some people like going to (2) ________ concerts, and listening to (3)

________ . The (4) ________ wear very formal clothes, and the (5) ________ is silent until the end of the (6)

________. Perhaps, you are a rock music (7) ________ . Rock concerts are often held at football (8) ________ or in

parks. (9) ________ of the audience dance to the music, or sing the songs. (10) ________ music is (11) ________ at

wedding and parties in many countries, and some people (12) ________ their own music at home. Nowadays, we (13)

________ music in shops and lifts, and many people (14) ________ their own music with them, or even (15)

________ to music when they study. Music is everywhere!

1. A. listen B. enjoy C. have D. preferring

2. A. classic B. classics C. classical D. classified

3. A. a group B. an orchestra C. a band D. a record

4. A. musicians B. actors C. musicals D. instruments

5. A. spectators B. people C. guests D. audience

6. A. happening B. action C. music D. performance

7. A. fan B. enthusiasm C. reader D. friend

8. A. matches B. grounds C. pitches D. pools

82

9. A. Members B. Selections C. Persons D. Those

10. A. Historical B. Nation C. Traditional D. Ancient

11. A. acted B. formed C. done D. played

12. A. do B. get C. make D. take

13. A. listen B. hear C. perform D. understand

14. A. carry B. wear C. lift D. play

15. A. hear B. have C. follow D. listen

SECTION IV: WRITING (5 points)

Part 1: Finish the second sentence in such a way that it means the same as the sentence before it beginning with

the words given. (2 points)

1. Despite Jack’s strange clothes, everybody ignored him.

Nobody took _______________________________________________________________________

2. It was more of a business arrangement than a marriage.

It was not _________________________________________________________________________

3. You must not communicate in any way with anyone involved in this case.

You must have ____________________________________________________________________

4. His father made him study Latin at the age of three.

At the age of three, he _______________________________________________________________

5. I would prefer you to pay in cash.

I’d rather _________________________________________________________________________

6. Try as I might, I couldn’t understand the code.

No matter _________________________________________________________________________

7. Professor Helsing knows everything about this menu script.

There is ___________________________________________________________________________

8. If you trade in your existing answer phone, we will reduce the price of a new one by $100.

Trade ____________________________________________________________________________

9. Norman was sorry because he had lost his temper.

Norman regretted ___________________________________________________________________

10. Although I respect the law, I cannot accept the court’s decision.

Much ____________________________________________________________________________

Part 2: Write a passage of about 150 words about a person that you love or admire most. (3 points)

83